Онлайн площадь треугольника по векторам: Онлайн калькулятор. Площадь треугольника построенного на векторах.

Содержание

Найти площадь треугольника по векторам онлайн. Векторное произведение

На данном уроке мы рассмотрим ещё две операции с векторами: векторное произведение векторов и смешанное произведение векторов (сразу ссылка, кому нужно именно оно) . Ничего страшного, так иногда бывает, что для полного счастья, помимо скалярного произведения векторов , требуется ещё и ещё. Такая вот векторная наркомания. Может сложиться впечатление, что мы залезаем в дебри аналитической геометрии. Это не так. В данном разделе высшей математики вообще мало дров, разве что на Буратино хватит. На самом деле материал очень распространенный и простой – вряд ли сложнее, чем то же скалярное произведение , даже типовых задач поменьше будет. Главное в аналитической геометрии, как многие убедятся или уже убедились, НЕ ОШИБАТЬСЯ В ВЫЧИСЛЕНИЯХ. Повторяйте как заклинание, и будет вам счастье =)

Если векторы сверкают где-то далеко, как молнии на горизонте, не беда, начните с урока Векторы для чайников , чтобы восстановить или вновь приобрести базовые знания о векторах. Более подготовленные читатели могут знакомиться с информацией выборочно, я постарался собрать максимально полную коллекцию примеров, которые часто встречаются в практических работах

Чем вас сразу порадовать? Когда я был маленьким, то умел жонглировать двумя и даже тремя шариками. Ловко получалось. Сейчас жонглировать не придётся вообще, поскольку мы будем рассматривать только пространственные векторы , а плоские векторы с двумя координатами останутся за бортом. Почему? Такими уж родились данные действия – векторное и смешанное произведение векторов определены и работают в трёхмерном пространстве. Уже проще!

В данной операции, точно так же, как и в скалярном произведении, участвуют два вектора . Пусть это будут нетленные буквы .

Само действие обозначается следующим образом: . Существуют и другие варианты, но я привык обозначать векторное произведение векторов именно так, в квадратных скобках с крестиком.

И сразу вопрос : если в скалярном произведении векторов участвуют два вектора, и здесь тоже умножаются два вектора, тогда в чём разница ? Явная разница, прежде всего, в РЕЗУЛЬТАТЕ:

Результатом скалярного произведения векторов является ЧИСЛО:

Результатом векторного произведения векторов является ВЕКТОР : , то есть умножаем векторы и получаем снова вектор. Закрытый клуб. Собственно, отсюда и название операции. В различной учебной литературе обозначения тоже могут варьироваться, я буду использовать букву .

Определение векторного произведения

Сначала будет определение с картинкой, затем комментарии.

Определение : Векторным произведением неколлинеарных векторов , взятых в данном порядке , называется ВЕКТОР , длина которого численно равна площади параллелограмма , построенного на данных векторах; вектор ортогонален векторам , и направлен так, что базис имеет правую ориентацию:

Разбираем определение по косточкам, тут много интересного!

Итак, можно выделить следующие существенные моменты:

1) Исходные векторы , обозначенные красными стрелками, по определению не коллинеарны . Случай коллинеарных векторов будет уместно рассмотреть чуть позже.

2) Векторы взяты в строго определённом порядке : – «а» умножается на «бэ» , а не «бэ» на «а». Результатом умножения векторов является ВЕКТОР , который обозначен синим цветом. Если векторы умножить в обратном порядке, то получим равный по длине и противоположный по направлению вектор (малиновый цвет). То есть, справедливо равенство .

3) Теперь познакомимся с геометрическим смыслом векторного произведения. Это очень важный пункт! ДЛИНА синего вектора (а, значит, и малинового вектора ) численно равна ПЛОЩАДИ параллелограмма, построенного на векторах . На рисунке данный параллелограмм заштрихован чёрным цветом.

Примечание : чертёж является схематическим, и, естественно, номинальная длина векторного произведения не равна площади параллелограмма.

Вспоминаем одну из геометрических формул: площадь параллелограмма равна произведению смежных сторон на синус угла между ними . Поэтому, исходя из вышесказанного, справедлива формула вычисления ДЛИНЫ векторного произведения:

Подчёркиваю, что в формуле речь идёт о ДЛИНЕ вектора, а не о самом векторе . Каков практический смысл? А смысл таков, что в задачах аналитической геометрии площадь параллелограмма часто находят через понятие векторного произведения:

Получим вторую важную формулу. Диагональ параллелограмма (красный пунктир) делит его на два равных треугольника. Следовательно, площадь треугольника, построенного на векторах (красная штриховка), можно найти по формуле:

4) Не менее важный факт состоит в том, что вектор ортогонален векторам , то есть . Разумеется, противоположно направленный вектор (малиновая стрелка) тоже ортогонален исходным векторам .

5) Вектор направлен так, что базис имеет правую ориентацию. На уроке о переходе к новому базису я достаточно подробно рассказал об ориентации плоскости , и сейчас мы разберёмся, что такое ориентация пространства. Объяснять буду на пальцах вашей правой руки . Мысленно совместите указательный палец с вектором и средний палец с вектором . Безымянный палец и мизинец прижмите к ладони. В результате большой палец – векторное произведение будет смотреть вверх. Это и есть правоориентированный базис (на рисунке именно он). Теперь поменяйте векторы (указательный и средний пальцы ) местами, в результате большой палец развернётся, и векторное произведение уже будет смотреть вниз. Это тоже правоориентированный базис. Возможно, у вас возник вопрос: а какой базис имеет левую ориентацию? «Присвойте» тем же пальцам левой руки векторы , и полУчите левый базис и левую ориентацию пространства

(в этом случае большой палец расположится по направлению нижнего вектора) . Образно говоря, данные базисы «закручивают» или ориентируют пространство в разные стороны. И это понятие не следует считать чем-то надуманным или абстрактным – так, например, ориентацию пространства меняет самое обычное зеркало, и если «вытащить отражённый объект из зазеркалья», то его в общем случае не удастся совместить с «оригиналом». Кстати, поднесите к зеркалу три пальца и проанализируйте отражение;-)

…как всё-таки хорошо, что вы теперь знаете о право- и левоориентированных базисах, ибо страшнЫ высказывания некоторых лекторов о смене ориентации =)

Векторное произведение коллинеарных векторов

Определение подробно разобрано, осталось выяснить, что происходит, когда векторы коллинеарны. Если векторы коллинеарны, то их можно расположить на одной прямой и наш параллелограмм тоже «складывается» в одну прямую. Площадь такого, как говорят математики,

вырожденного параллелограмма равна нулю. Это же следует и из формулы – синус нуля или 180-ти градусов равен нулю, а значит, и площадь нулевая

Таким образом, если , то и . Обратите внимание, что само векторное произведение равно нулевому вектору, но на практике этим часто пренебрегают и пишут, что оно тоже равно нулю.

Частный случай – векторное произведение вектора на самого себя:

С помощью векторного произведения можно проверять коллинеарность трёхмерных векторов, и данную задачу среди прочих мы тоже разберём.

Для решения практических примеров может потребоваться тригонометрическая таблица , чтобы находить по ней значения синусов.

Ну что же, разжигаем огонь:

Пример 1

а) Найти длину векторного произведения векторов , если

б) Найти площадь параллелограмма, построенного на векторах , если

Решение : Нет, это не опечатка, исходные данные в пунктах условия я намеренно сделал одинаковыми. Потому что оформление решений будет отличаться!

а) По условию требуется найти длину вектора (векторного произведения). По соответствующей формуле:

Ответ :

Коль скоро спрашивалось о длине, то в ответе указываем размерность – единицы.

б) По условию требуется найти площадь параллелограмма, построенного на векторах . Площадь данного параллелограмма численно равна длине векторного произведения:

Ответ :

Обратите внимание, что в ответе о векторном произведении речи не идёт вообще, нас спрашивали о площади фигуры , соответственно, размерность – квадратные единицы.

Всегда смотрим, ЧТО требуется найти по условию, и, исходя из этого, формулируем чёткий ответ. Может показаться буквоедством, но буквоедов среди преподавателей хватает, и задание с хорошими шансами вернётся на доработку. Хотя это не особо натянутая придирка – если ответ некорректен, то складывается впечатление, что человек не разбирается в простых вещах и/или не вник в суть задания. Этот момент всегда нужно держать на контроле, решая любую задачу по высшей математике, да и по другим предметам тоже.

Куда подевалась большая буковка «эн»? В принципе, её можно было дополнительно прилепить в решение, но в целях сократить запись, я этого не сделал. Надеюсь, всем понятно, что и – это обозначение одного и того же.

Популярный пример для самостоятельного решения:

Пример 2

Найти площадь треугольника, построенного на векторах , если

Формула нахождения площади треугольника через векторное произведение дана в комментариях к определению. Решение и ответ в конце урока.

На практике задача действительно очень распространена, треугольниками вообще могут замучить.

Для решения других задач нам понадобятся:

Свойства векторного произведения векторов

Некоторые свойства векторного произведения мы уже рассмотрели, тем не менее, я их включу в данный список.

Для произвольных векторов и произвольного числа справедливы следующие свойства:

1) В других источниках информации данный пункт обычно не выделяют в свойствах, но он очень важен в практическом плане. Поэтому пусть будет.

2) – свойство тоже разобрано выше, иногда его называют антикоммутативностью . Иными словами, порядок векторов имеет значение.

3) – сочетательные или ассоциативные законы векторного произведения. Константы безпроблемно выносятся за пределы векторного произведения. Действительно, чего им там делать?

4) – распределительные или дистрибутивные законы векторного произведения. С раскрытием скобок тоже нет проблем.

В качестве демонстрации рассмотрим коротенький пример:

Пример 3

Найти , если

Решение: По условию снова требуется найти длину векторного произведения. Распишем нашу миниатюру:

(1) Согласно ассоциативным законам, выносим константы за переделы векторного произведения.

(2) Выносим константу за пределы модуля, при этом модуль «съедает» знак «минус». Длина же не может быть отрицательной.

(3) Дальнейшее понятно.

Ответ :

Пора подбросить дров в огонь:

Пример 4

Вычислить площадь треугольника, построенного на векторах , если

Решение : Площадь треугольника найдём по формуле . Загвоздка состоит в том, что векторы «цэ» и «дэ» сами представлены в виде сумм векторов. Алгоритм здесь стандартен и чем-то напоминает примеры № 3 и 4 урока Скалярное произведение векторов . Решение для ясности разобьём на три этапа:

1) На первом шаге выразим векторное произведение через векторное произведение , по сути, выразим вектор через вектор . О длинах пока ни слова!

(1) Подставляем выражения векторов .

(2) Используя дистрибутивные законы, раскрываем скобки по правилу умножения многочленов.

(3) Используя ассоциативные законы, выносим все константы за пределы векторных произведений. При маломальском опыте действия 2 и 3 можно выполнять одновременно.

(4) Первое и последнее слагаемое равно нулю (нулевому вектору) благодаря приятному свойству . Во втором слагаемом используем свойство антикоммутативности векторного произведения:

(5) Приводим подобные слагаемые.

В результате вектор оказался выражен через вектор, чего и требовалось достичь:

2) На втором шаге найдем длину нужного нам векторного произведения. Данное действие напоминает Пример 3:

3) Найдём площадь искомого треугольника:

Этапы 2-3 решения можно было оформить и одной строкой.

Ответ :

Рассмотренная задача достаточно распространена в контрольных работах, вот пример для самостоятельного решения:

Пример 5

Найти , если

Краткое решение и ответ в конце урока. Посмотрим, насколько вы были внимательны при изучении предыдущих примеров;-)

Векторное произведение векторов в координатах , заданных в ортонормированном базисе , выражается формулой :

Формула и правда простецкая: в верхнюю строку определителя записываем координатные векторы, во вторую и третью строки «укладываем» координаты векторов , причём укладываем в строгом порядке – сначала координаты вектора «вэ», затем координаты вектора «дубль-вэ». Если векторы нужно умножить в другом порядке, то и строки следует поменять местами:

Пример 10

Проверить, будут ли коллинеарны следующие векторы пространства:
а)
б)

Решение : Проверка основана на одном из утверждений данного урока: если векторы коллинеарны, то их векторное произведение равно нулю (нулевому вектору): .

а) Найдём векторное произведение:

Таким образом, векторы не коллинеарны.

б) Найдём векторное произведение:

Ответ : а) не коллинеарны, б)

Вот, пожалуй, и все основные сведения о векторном произведении векторов.

Данный раздел будет не очень большим, так как задач, где используется смешанное произведение векторов, немного. Фактически всё будет упираться в определение, геометрический смысл и пару рабочих формул.

Смешанное произведение векторов – это произведение трёх векторов :

Вот так вот они выстроились паровозиком и ждут, не дождутся, когда их вычислят.

Сначала опять определение и картинка:

Определение : Смешанным произведением некомпланарных векторов , взятых в данном порядке , называется объём параллелепипеда , построенного на данных векторах, снабжённый знаком «+», если базис правый, и знаком «–», если базис левый.

Выполним рисунок. Невидимые нам линии прочерчены пунктиром:

Погружаемся в определение:

2) Векторы взяты в определённом порядке , то есть перестановка векторов в произведении , как вы догадываетесь, не проходит без последствий.

3) Перед тем, как прокомментировать геометрический смысл, отмечу очевидный факт: смешанное произведение векторов является ЧИСЛОМ : . В учебной литературе оформление может быть несколько другим, я привык обозначать смешанное произведение через , а результат вычислений буквой «пэ».

По определению смешанное произведение – это объем параллелепипеда , построенного на векторах (фигура прочерчена красными векторами и линиями чёрного цвета). То есть, число равно объему данного параллелепипеда.

Примечание : чертёж является схематическим.

4) Не будем заново париться с понятием ориентации базиса и пространства. Смысл заключительной части состоит в том, что к объёму может добавляться знак минус. Простыми словами, смешанное произведение может быть отрицательным: .

Непосредственно из определения следует формула вычисления объема параллелепипеда, построенного на векторах .

Контрольная работа №1

Векторы. Элементы высшей алгебры

1-20. Известны длины векторов и и; – угол между этими векторами.

Вычислить: 1) и, 2) .3) Найти площадь треугольника, построенного на векторах и.

Сделать чертеж.

Решение. Используя определение скалярного произведения векторов:

И свойства скалярного произведения: ,

1) находим скалярный квадрат вектора:

то есть, Тогда .

Рассуждая аналогично, получаем

то есть, Тогда .

По определению векторного произведения: ,

с учетом того, что

Площадь треугольника построенного на векторах и равна

21-40. Известны координаты трех вершин A, B, D параллелограмма ABCD . Средствами векторной алгебры требуется:

A (3;0;-7), B (2;4;6), D (-7;-5;1)

Решение.

Известно, что диагонали параллелограмма в точке пересечения делятся пополам. Поэтому координаты точки E — пересечения диагоналей — найдем как координаты середины отрезка BD . Обозначая их через x E ,y E , z E получим, что

Получаем .

Зная координаты точки E — середины диагонали BD и координаты одного из его концов A (3;0;-7), по формулам определяем искомые координаты вершины С параллелограмма:

Итак, вершина .

2) Чтобы найти проекцию вектора на вектор , найдем координаты этих векторов: ,

аналогично . Проекцию вектора на вектор , находим по формуле:

3) Угол между диагоналями параллелограмма находим как угол между векторами

И по свойству скалярного произведения:

тогда

4) Площадь параллелограмма находим как модуль векторного произведения:

5) Объем пирамиды находим как одну шестую модуля смешанного произведения векторов , где О(0;0;0), тогда

Тогда искомый объем (куб.ед.)

41-60. Даны матрицы:

В ·С -1 +3A T

Обозначения:

Сначала находим обратную матрицу к матрице С.

Для этого находим ее определитель:

Определитель отличен от нуля, следовательно, матрица является невырожденной и для нее можно найти обратную матрицу С -1

Найдем алгебраические дополнения по формуле , где — минор элемента :

Тогда , .

61–80. Решите систему линейных уравнений:

    Методом Крамера; 2. Матричным методом.

Решение.

а) метод Крамера

Найдем определитель системы

Так как , то система имеет единственное решение.

Найдем определители и , заменив в матрице коэффициентов соответственно первый, второй, третий столбцы столбцом свободных членов.

По формулам Крамера:

б) матричный метод (с помощью обратной матрицы).

Данную систему запишем в матричной форме и решим с помощью обратной матрицы.

Пусть А – матрица коэффициентов при неизвестных; X – матрица-столбец неизвестных x , y , z и Н – матрица-столбец из свободных членов:

Левую часть системы (1) можно записать в виде произведения матриц , а правую в виде матрицы Н . Следовательно имеем матричное уравнение

Так как определитель матрицы А отличен от нуля (пункт «а»), то матрица А имеет обратную матрицу . Умножим обе части равенства (2) слева на матрицу , получим

Так как , где Е – единичная матрица, а , то

Пусть имеем невырожденную матрицу А:

Тогда обратную матрицу находим по формуле:

где A ij — алгебраическое дополнение элемента a ij в определителе матрицы А , которое является произведением (-1) i+j на минор (определитель) n-1 порядка, полученный вычеркиванием i-й строки и j-го столбца в определителе матрицы А:

Отсюда получаем обратную матрицу:

Столбец X: X=A -1 H

81–100. Решить систему линейных уравнений методом Гаусса

Решение. Запишем систему в виде расширенной матрицы:

Выполняем элементарные преобразования со строками.

Из 2-ой строки вычитаем первую строку, умноженную на 2. Из строки 3 вычитаем первую строку, умноженную на 4. Из строки 4 вычитаем первую строку, получаем матрицу:

Далее получаем нуль в первом столбце последующих строк, для этого из второй строки вычитаем третью строку. Из третьей строки вычитаем вторую строку, умноженную на 2. Из четвертой строки вычитаем вторую строку, умноженную на 3. В результате получаем матрицу вида:

Из четвертой строки вычитаем третью.

Поменяем местами предпоследнюю и последнюю строки:

Последняя матрица равносильна системе уравнений:

Из последнего уравнения системы находим .

Подставляя в предпоследнее уравнение, получаем .

Из второго уравнения системы следует, что

Из первого уравнения находим х:

Ответ:

Контрольная работа №2

Аналитическая геометрия

1-20. Даны координаты вершин треугольника АВС. Найти:

1) длину стороны A В ;

2) уравнения сторон АВ и ВС и их угловые коэффициенты;

3) угол В в радианах с точностью до двух знаков;

4) уравнение высоты CD и её длину;

5) уравнение медианы АЕ

высотой CD ;

К параллельно стороне АВ,

7) сделать чертёж.

А(3;6), В(15;-3), С(13;11)

Решение.

Применяя (1), находим длину стороны АВ :

2) уравнения сторон АВ и ВС и их угловые коэффициенты:

Уравнение прямой, проходящей через точки и , имеет вид

Подставляя в (2) координаты точек А и В , получим уравнение стороны АВ :

(АВ ).

(BC ).

3) угол В в радианах с точностью до двух знаков.

Известно, что тангенс угла между двумя прямыми, угловые коэффициенты, которых соответственно равны и вычисляется по формуле

Искомый угол В образован прямыми АВ и ВС , угловые коэффициенты которых найдены: ; . Применяя (3), получим

; , или

4) уравнение высоты CD и её длина.

Расстояние от точки С до прямой АВ:

5) уравнение медианы АЕ и координаты точки К пересечения этой медианы с

высотой CD .

середина стороны ВС:

Тогда уравнение АЕ:

Решаем систему уравнений:

6) уравнение прямой, проходящей через точку К параллельно стороне АВ :

Так как искомая прямая параллельна стороне АВ , то ее угловой коэффициент будет равен угловому коэффициенту прямой АВ . Подставив в (4) координаты найденной точки К и угловой коэффициент , получим

; (KF ).

Площадь параллелограмма равна 12 кв. ед., две его вершины – точкиА(-1;3) и В(-2;4). Найти две другие вершины этого параллелограмма, если известно, что точка пересечения его диагоналей лежит на оси абсцисс. Сделать чертёж.

Решение. Пусть точка пересечения диагоналей имеет координаты .

Тогда очевидно, что и

следовательно, координаты векторов .

Площадь параллелограмма находим по формуле

Тогда координаты двух других вершин .

В задачах 51-60 даны координаты точек А и В . Требуется:

    Составить каноническое уравнение гиперболы, проходящей через данные точкиА и В, если фокусы гиперболы расположены на оси абсцисс;

    Найти полуоси, фокусы, эксцентриситет и уравнения асимптот этой гиперболы;

    Найти все точки пересечения гиперболы с окружностью с центром в начале координат, если эта окружность проходит через фокусы гиперболы;

    Построить гиперболу, её асимптоты и окружность.

А(6;-2), В(-8;12).

Решение. Уравнение искомой гиперболы в каноническом виде записывается

где a — действительная полуось гиперболы, b — мнимая полуось. Подставляя координаты точек А и В в это уравнение найдем эти полуоси:

– уравнение гиперболы: .

Полуоси а=4,

фокусное расстояние Фокусы (-8,0) и (8,0)

Эксцентриситет

Асиптоты:

Если окружность проходит через начало координат, ее уравнение

Подставляя один из фокусов, находим и уравнение окружности

Находим точки пересечения гиперболы и окружности:

Строим чертеж:

В задачах 61-80 построить график функции в полярной системе координат по точкам, придавая  значения через промежуток /8 (0 2). Найти уравнение линии в прямоугольной декартовой системе координат (положительная полуось абсцисс совпадает с полярной осью, а полюс – с началом координат).

Решение. Построим линию по точкам, предварительно заполнив таблицу значений и φ.

Номер

φ ,

φ, градусы

Номер

φ , рад

градусы

3∙(x 2 +2∙1x + 1) -3∙1 = 3(x+1) 2 — 3

делаем вывод, что данное уравнение определяет эллипс:

Даны точки А, В, С, D. Требуется найти:

1. Уравнение плоскости(Q ), проходящей через точкиА, В, С D в плоскости (Q) ;

2. Уравнение прямой (I), проходящей через точкиВ и D;

3. Угол между плоскостью (Q) и прямой (I) ;

4. Уравнение плоскости (Р), проходящей через точкуА перпендикулярно прямой (I) ;

5. Угол между плоскостями (Р) и (Q ) ;

6. Уравнение прямой (т), проходящей через точку А в направлении ее радиус-вектора;

7. Угол между прямыми (I) и (т).

А(9;-8;1), В(-9;4;5), С(9;-5;5), D (6;4;0)

1. Уравнение плоскости(Q ), проходящей через точки А, В, С и проверить, лежит ли точка D в плоскости определяется по формуле Найти : 1) . 2) Площадь параллелограмма, построенного на и. 3) Объем параллелепипеда, построенного на векторах , и. Контрольная работа по теме «Элементы теории линейных пространств…

  • Методические рекомендации по выполнению контрольных работ для бакалавриата заочной формы обучения по квалификации 080100. 62 по направлению

    Методические рекомендации

    Параллелепипеда и объем пирамиды, построенных на векторах , и. Решение: 2-=2(1;1;1)-(2;1;4)= (2;2;2)-(2;1;4)=(0;1;-2).. . . . . 4.ЗАДАЧИ ДЛЯ КОНТРОЛЬНЫХ РАБОТ Раздел I. Линейная алгебра . 1 – 10. Дана…

  • В этой статье мы подробно остановимся на понятии векторного произведения двух векторов. Мы дадим необходимые определения, запишем формулу для нахождения координат векторного произведения, перечислим и обоснуем его свойства. После этого остановимся на геометрическом смысле векторного произведения двух векторов и рассмотрим решения различных характерных примеров.

    Навигация по странице.

    Определение векторного произведения.

    Прежде чем дать определение векторного произведения, разберемся с ориентацией упорядоченной тройки векторов в трехмерном пространстве.

    Отложим векторы от одной точки. В зависимости от направления вектора тройка может быть правой или левой. Посмотрим с конца вектора на то, как происходит кратчайший поворот от вектора к . Если кратчайший поворот происходит против часовой стрелки, то тройка векторов называется правой , в противном случае – левой .


    Теперь возьмем два не коллинеарных вектора и . Отложим от точки А векторы и . Построим некоторый вектор , перпендикулярный одновременно и и . Очевидно, что при построении вектора мы можем поступить двояко, задав ему либо одно направление, либо противоположное (смотрите иллюстрацию).


    В зависимости от направления вектора упорядоченная тройка векторов может быть правой или левой.

    Так мы вплотную подошли к определению векторного произведения. Оно дается для двух векторов, заданных в прямоугольной системе координат трехмерного пространства.

    Определение.

    Векторным произведением двух векторов и , заданных в прямоугольной системе координат трехмерного пространства, называется такой вектор , что

    Векторное произведение векторов и обозначается как .

    Координаты векторного произведения.

    Сейчас дадим второе определение векторного произведения, которое позволяет находить его координаты по координатам заданных векторов и.

    Определение.

    В прямоугольной системе координат трехмерного пространства векторное произведение двух векторов и есть вектор , где — координатные векторы.

    Это определение дает нам векторное произведение в координатной форме.

    Векторное произведение удобно представлять в виде определителя квадратной матрицы третьего порядка, первая строка которой есть орты , во второй строке находятся координаты вектора , а в третьей – координаты вектора в заданной прямоугольной системе координат:

    Если разложить этот определитель по элементам первой строки, то получим равенство из определения векторного произведения в координатах (при необходимости обращайтесь к статье ):

    Следует отметить, что координатная форма векторного произведения полностью согласуется с определением, данным в первом пункте этой статьи. Более того, эти два определения векторного произведения эквивалентны. Доказательство этого факта можете посмотреть в книге, указанной в конце статьи.

    Свойства векторного произведения.

    Так как векторное произведение в координатах представимо в виде определителя матрицы , то на основании легко обосновываются следующие свойства векторного произведения :

    Для примера докажем свойство антикоммутативности векторного произведения.

    По определению и . Нам известно, что значение определителя матрицы изменяется на противоположное, если переставить местами две строки, поэтому, , что доказывает свойство антикоммутативности векторного произведения.

    Векторное произведение – примеры и решения.

    В основном встречаются три типа задач.

    В задачах первого типа заданы длины двух векторов и угол между ними, а требуется найти длину векторного произведения. В этом случае используется формула .

    Пример.

    Найдите длину векторного произведения векторов и , если известно .

    Решение.

    Мы знаем из определения, что длина векторного произведения векторов и равна произведению длин векторов и на синус угла между ними, поэтому, .

    Ответ:

    .

    Задачи второго типа связаны с координатами векторов, в них векторное произведение, его длина или что-либо еще ищется через координаты заданных векторов и .

    Здесь возможна масса различных вариантов. К примеру, могут быть заданы не координаты векторов и , а их разложения по координатным векторам вида и , или векторы и могут быть заданы координатами точек их начала и конца.

    Рассмотрим характерные примеры.

    Пример.

    В прямоугольной системе координат заданы два вектора . Найдите их векторное произведение.

    Решение.

    По второму определению векторное произведение двух векторов в координатах записывается как:

    К такому же результату мы бы пришли, если бы векторное произведение записали через определитель

    Ответ:

    .

    Пример.

    Найдите длину векторного произведения векторов и , где — орты прямоугольной декартовой системы координат.

    Решение.

    Сначала найдем координаты векторного произведения в заданной прямоугольной системе координат.

    Так как векторы и имеют координаты и соответственно (при необходимости смотрите статью координаты вектора в прямоугольной системе координат), то по второму определению векторного произведения имеем

    То есть, векторное произведение имеет координаты в заданной системе координат.

    Длину векторного произведения находим как корень квадратный из суммы квадратов его координат (эту формулу длины вектора мы получили в разделе нахождение длины вектора):

    Ответ:

    .

    Пример.

    В прямоугольной декартовой системе координат заданы координаты трех точек . Найдите какой-нибудь вектор, перпендикулярный и одновременно.

    Решение.

    Векторы и имеют координаты и соответственно (смотрите статью нахождение координат вектора через координаты точек). Если найти векторное произведение векторов и , то оно по определению является вектором, перпендикулярным и к и к , то есть, является решением нашей задачи. Найдем его

    Ответ:

    — один из перпендикулярных векторов.

    В задачах третьего типа проверяется навык использования свойств векторного произведения векторов. После применения свойств, применяются соответствующие формулы.

    Пример.

    Векторы и перпендикулярны и их длины равны соответственно 3 и 4 . Найдите длину векторного произведения .

    Решение.

    По свойству дистрибутивности векторного произведения мы можем записать

    В силу сочетательного свойства вынесем числовые коэффициенты за знак векторных произведений в последнем выражении:

    Векторные произведения и равны нулю, так как и , тогда .

    Так как векторное произведение антикоммутативно, то .

    Итак, с помощью свойств векторного произведения мы пришли к равенству .

    По условию векторы и перпендикулярны, то есть угол между ними равен . То есть, у нас есть все данные для нахождения требуемой длины

    Ответ:

    .

    Геометрический смысл векторного произведения.

    По определению длина векторного произведения векторов равна . А из курса геометрии средней школы нам известно, что площадь треугольника равна половине произведения длин двух сторон треугольника на синус угла между ними. Следовательно, длина векторного произведения равна удвоенной площади треугольника, имеющего сторонами векторы и , если их отложить от одной точки. Другими словами, длина векторного произведения векторов и равна площади параллелограмма со сторонами и и углом между ними, равным . В этом состоит геометрический смысл векторного произведения.

    Онлайн калькулятор. Векторное произведение векторов

    Смешанным (или векторно-скалярным) произведением трех векторов a, b, c (взятых в указанном порядке) называется скалярное произведение вектора a на векторное произведение b x c , т. е. число a(b x c), или, что то же, (b x c)a.
    Обозначение: abc .

    Назначение . Онлайн-калькулятор предназначен для вычисления смешанного произведения векторов. Полученное решение сохраняется в файле Word . Дополнительно создается шаблон решения в Excel .

    Признаки компланарности векторов

    Три вектора (или большее число) называются компланарными, если они, будучи приведены к общему началу, лежат в одной плоскости.
    Если хотя бы один из трех векторов – нулевой, то три вектора тоже считаются компланарными.

    Признак компланарности . Если система a, b, c – правая, то abc>0 ; если левая, то abcГеометрический смысл смешанного произведения . Смешанное произведение abc трех некомпланарных векторов a, b, c равно объему параллелепипеда, построенного на векторах a, b, c , взятому со знаком плюс, если система a, b, c – правая, и со знаком минус, если эта система левая.

    Свойства смешанного произведения

    1. При круговой перестановке сомножителей смешанное произведение не меняется, при перестановке двух сомножителей – меняет знак на обратный: abc=bca=cab=-(bac)=-(cba)=-(acb)
      Вытекает из геометрического смысла.
    2. (a+b)cd=acd+bcd (распределительное свойство). Распространяется на любое число слагаемых.
      Вытекает из определения смешанного произведения.
    3. (ma)bc=m(abc) (сочетательное свойство относительно скалярного множителя).
      Вытекает из определения смешанного произведения. Эти свойства позволяют применять к смешанным произведениям преобразования, отличающиеся от обычных алгебраических лишь тем, что менять порядок сомножителей можно только с учетом знака произведения.
    4. Смешанное произведение, имеющее хотя бы два равных сомножителя, равно нулю: aab=0 .

    Пример №1 . Найти смешанное произведение. ab(3a+2b-5c)=3aba+2abb-5abc=-5abc .

    Пример №2 . (a+b)(b+c)(c+a)= (axb+axc+bxb+bxc)(c+a)= (axb+axc +bxc)(c+a)=abc+acc+aca+aba+bcc+bca . Все члены, кроме двух крайних, равны нулю. Кроме того, bca=abc . Поэтому (a+b)(b+c)(c+a)=2abc .

    Пример №3 . Вычислить смешанное произведение трех векторов a=15i+20j+5k, b=2i-4j+14k, c=3i-6j+21k .
    Решение . Чтобы вычислить смешанное произведение векторов, необходимо найти определитель системы, составленной из координат векторов. Запишем систему в виде.

    Данный онлайн калькулятор вычисляет смешанное произведение векторов. Дается подробное решение. Для вычисления смешанного произведения векторов выберите способ представления векторов (по координатам или по двум точкам) введите данные в ячейки и нажимайте на кнопку «Вычислить.»

    ×

    Предупреждение

    Очистить все ячейки?

    Закрыть Очистить

    Инструкция ввода данных. Числа вводятся в виде целых чисел (примеры: 487, 5, -7623 и т.д.), десятичных чисел (напр. 67., 102.54 и т.д.) или дробей. Дробь нужно набирать в виде a/b, где a и b (b>0) целые или десятичные числа. Примеры 45/5, 6.6/76.4, -7/6.7 и т.д.

    Смешанное произведение векторов (теория)

    Смешанное произведение трех векторов это число, которое получается при скалярном произведении результата векторного произведения первых двух векторов на третьий вектор. Другими словами, если заданы три вектора a, b и c , то для получения смешанного произведения этих векторов, сначала векторно умножаются первые два вектора и полученный вектор [ab ] скалярно умножается на вектор c .

    Смешанное произведение трех векторов a, b и c обозначается так: abc или так (a,b,c ). Тогда можно записать:

    Прежде чем сформулировать теорему, представляющую геометрический смысл смешанного произведения, ознакомьтесь с понятиями правая тройка, левая тройка, правая система координат, левая система координат (определения 2, 2″ и 3 на странице векторное произведение векторов онлайн).

    Для определенности, в дальнейшем мы будем рассматривать только правые системы координат.

    Теорема 1. Смешанное произведение векторов ([ab ],c ) равно объему параллелипеда, построенного на приведенных к общему началу векторах a, b, c , взятому со знаком плюс, если тройка a, b, c правая, и со знаком минус, если тройка a, b, c левая. Если векторы a, b, c компланарны, то ([ab ],c ) равно нулю.

    Следствие 1. Имеет место следующее равенство:

    Следовательно нам достаточно доказать, что

    ([ab ],c )=([bc ],a )(3)

    Из выражения (3) видно, что левая и правая часть равны объему параллелипеда. Но и знаки правой и левой частей совпадают, так как тройки векторов abc и bca имеют одинаковую ориентацию.

    Доказанное равенство (1) позволяет записать смешанное произведение трех векторов a, b, c просто в виде abc , не указывая, какие именно два вектора перемножаются векторно первые два или последние два.

    Следствие 2. Необходимым и достаточным условием компланарности трех векторов является равенство нулю их смешанного произведения.

    Доказательство вытекает из теоремы 1. Действительно, если векторы компланарны, то смешанное произведение этих векторов равно нулю. Обратное, если смешанное произведение равно нулю, то из теоремы 1 вытекает компланарность этих векторов (так как объем параллелипеда, построенного на приведенных к общему началу векторах равно нулю).

    Следствие 3. Смешанное произведение трех векторов, два из которых совпадают, равно нулю.

    Действительно. Если два вектора из трех совпадают, то они компланарны. Следовательно, смешанное произведение этих векторов равно нулю.

    Смешанное произведение векторов в декартовых координатах

    Теорема 2. Пусть три вектора a, b и c определены своими декартовыми прямоугольными координатами

    Доказательство. Смешанное произведение abc равно скалярному произведению векторов [ab ] и c . Векторное произведение векторов [ab ] в декартовых координатах вычисляется формулой ():

    Последнее выражение можно записать, используя определители второго порядка:

    необходимо и достаточно равенство нулю определителя, строки которой заполнены координатами этих векторов, т.е:

    .(7)

    Для доказательства следствия достаточно рассмотреть формулу (4) и следствие 2.

    Смешанное произведение векторов на примерах

    Пример 1. Найти смешанное произведение векторов abс , где

    Смешанное произведение векторов a, b, c равен определителю матрицы L . Вычислим определитель матрицы L , разложив определитель по строке 1:

    Конечная точка вектора a .

    На данном уроке мы рассмотрим ещё две операции с векторами: векторное произведение векторов и смешанное произведение векторов (сразу ссылка, кому нужно именно оно) . Ничего страшного, так иногда бывает, что для полного счастья, помимо скалярного произведения векторов , требуется ещё и ещё. Такая вот векторная наркомания. Может сложиться впечатление, что мы залезаем в дебри аналитической геометрии. Это не так. В данном разделе высшей математики вообще мало дров, разве что на Буратино хватит. На самом деле материал очень распространенный и простой – вряд ли сложнее, чем то же скалярное произведение , даже типовых задач поменьше будет. Главное в аналитической геометрии, как многие убедятся или уже убедились, НЕ ОШИБАТЬСЯ В ВЫЧИСЛЕНИЯХ. Повторяйте как заклинание, и будет вам счастье =)

    Если векторы сверкают где-то далеко, как молнии на горизонте, не беда, начните с урока Векторы для чайников , чтобы восстановить или вновь приобрести базовые знания о векторах. Более подготовленные читатели могут знакомиться с информацией выборочно, я постарался собрать максимально полную коллекцию примеров, которые часто встречаются в практических работах

    Чем вас сразу порадовать? Когда я был маленьким, то умел жонглировать двумя и даже тремя шариками. Ловко получалось. Сейчас жонглировать не придётся вообще, поскольку мы будем рассматривать только пространственные векторы , а плоские векторы с двумя координатами останутся за бортом. Почему? Такими уж родились данные действия – векторное и смешанное произведение векторов определены и работают в трёхмерном пространстве. Уже проще!

    В данной операции, точно так же, как и в скалярном произведении, участвуют два вектора . Пусть это будут нетленные буквы .

    Само действие обозначается следующим образом: . Существуют и другие варианты, но я привык обозначать векторное произведение векторов именно так, в квадратных скобках с крестиком.

    И сразу вопрос : если в скалярном произведении векторов участвуют два вектора, и здесь тоже умножаются два вектора, тогда в чём разница ? Явная разница, прежде всего, в РЕЗУЛЬТАТЕ:

    Результатом скалярного произведения векторов является ЧИСЛО:

    Результатом векторного произведения векторов является ВЕКТОР : , то есть умножаем векторы и получаем снова вектор. Закрытый клуб. Собственно, отсюда и название операции. В различной учебной литературе обозначения тоже могут варьироваться, я буду использовать букву .

    Определение векторного произведения

    Сначала будет определение с картинкой, затем комментарии.

    Определение : Векторным произведением неколлинеарных векторов , взятых в данном порядке , называется ВЕКТОР , длина которого численно равна площади параллелограмма , построенного на данных векторах; вектор ортогонален векторам , и направлен так, что базис имеет правую ориентацию:

    Разбираем определение по косточкам, тут много интересного!

    Итак, можно выделить следующие существенные моменты:

    1) Исходные векторы , обозначенные красными стрелками, по определению не коллинеарны . Случай коллинеарных векторов будет уместно рассмотреть чуть позже.

    2) Векторы взяты в строго определённом порядке : – «а» умножается на «бэ» , а не «бэ» на «а». Результатом умножения векторов является ВЕКТОР , который обозначен синим цветом. Если векторы умножить в обратном порядке, то получим равный по длине и противоположный по направлению вектор (малиновый цвет). То есть, справедливо равенство .

    3) Теперь познакомимся с геометрическим смыслом векторного произведения. Это очень важный пункт! ДЛИНА синего вектора (а, значит, и малинового вектора ) численно равна ПЛОЩАДИ параллелограмма, построенного на векторах . На рисунке данный параллелограмм заштрихован чёрным цветом.

    Примечание : чертёж является схематическим, и, естественно, номинальная длина векторного произведения не равна площади параллелограмма.

    Вспоминаем одну из геометрических формул: площадь параллелограмма равна произведению смежных сторон на синус угла между ними . Поэтому, исходя из вышесказанного, справедлива формула вычисления ДЛИНЫ векторного произведения:

    Подчёркиваю, что в формуле речь идёт о ДЛИНЕ вектора, а не о самом векторе . Каков практический смысл? А смысл таков, что в задачах аналитической геометрии площадь параллелограмма часто находят через понятие векторного произведения:

    Получим вторую важную формулу. Диагональ параллелограмма (красный пунктир) делит его на два равных треугольника. Следовательно, площадь треугольника, построенного на векторах (красная штриховка), можно найти по формуле:

    4) Не менее важный факт состоит в том, что вектор ортогонален векторам , то есть . Разумеется, противоположно направленный вектор (малиновая стрелка) тоже ортогонален исходным векторам .

    5) Вектор направлен так, что базис имеет правую ориентацию. На уроке о переходе к новому базису я достаточно подробно рассказал об ориентации плоскости , и сейчас мы разберёмся, что такое ориентация пространства. Объяснять буду на пальцах вашей правой руки . Мысленно совместите указательный палец с вектором и средний палец с вектором . Безымянный палец и мизинец прижмите к ладони. В результате большой палец – векторное произведение будет смотреть вверх. Это и есть правоориентированный базис (на рисунке именно он). Теперь поменяйте векторы (указательный и средний пальцы ) местами, в результате большой палец развернётся, и векторное произведение уже будет смотреть вниз. Это тоже правоориентированный базис. Возможно, у вас возник вопрос: а какой базис имеет левую ориентацию? «Присвойте» тем же пальцам левой руки векторы , и полУчите левый базис и левую ориентацию пространства (в этом случае большой палец расположится по направлению нижнего вектора) . Образно говоря, данные базисы «закручивают» или ориентируют пространство в разные стороны. И это понятие не следует считать чем-то надуманным или абстрактным – так, например, ориентацию пространства меняет самое обычное зеркало, и если «вытащить отражённый объект из зазеркалья», то его в общем случае не удастся совместить с «оригиналом». Кстати, поднесите к зеркалу три пальца и проанализируйте отражение;-)

    …как всё-таки хорошо, что вы теперь знаете о право- и левоориентированных базисах, ибо страшнЫ высказывания некоторых лекторов о смене ориентации =)

    Векторное произведение коллинеарных векторов

    Определение подробно разобрано, осталось выяснить, что происходит, когда векторы коллинеарны. Если векторы коллинеарны, то их можно расположить на одной прямой и наш параллелограмм тоже «складывается» в одну прямую. Площадь такого, как говорят математики, вырожденного параллелограмма равна нулю. Это же следует и из формулы – синус нуля или 180-ти градусов равен нулю, а значит, и площадь нулевая

    Таким образом, если , то . Строго говоря, само векторное произведение равно нулевому вектору, но на практике этим часто пренебрегают и пишут, что оно просто равно нулю.

    Частный случай – векторное произведение вектора на самого себя:

    С помощью векторного произведения можно проверять коллинеарность трёхмерных векторов, и данную задачу среди прочих мы тоже разберём.

    Для решения практических примеров может потребоваться тригонометрическая таблица , чтобы находить по ней значения синусов.

    Ну что же, разжигаем огонь:

    Пример 1

    а) Найти длину векторного произведения векторов , если

    б) Найти площадь параллелограмма, построенного на векторах , если

    Решение : Нет, это не опечатка, исходные данные в пунктах условия я намеренно сделал одинаковыми. Потому что оформление решений будет отличаться!

    а) По условию требуется найти длину вектора (векторного произведения). По соответствующей формуле:

    Ответ :

    Коль скоро спрашивалось о длине, то в ответе указываем размерность – единицы.

    б) По условию требуется найти площадь параллелограмма, построенного на векторах . Площадь данного параллелограмма численно равна длине векторного произведения:

    Ответ :

    Обратите внимание, что в ответе о векторном произведении речи не идёт вообще, нас спрашивали о площади фигуры , соответственно, размерность – квадратные единицы.

    Всегда смотрим, ЧТО требуется найти по условию, и, исходя из этого, формулируем чёткий ответ. Может показаться буквоедством, но буквоедов среди преподавателей хватает, и задание с хорошими шансами вернётся на доработку. Хотя это не особо натянутая придирка – если ответ некорректен, то складывается впечатление, что человек не разбирается в простых вещах и/или не вник в суть задания. Этот момент всегда нужно держать на контроле, решая любую задачу по высшей математике, да и по другим предметам тоже.

    Куда подевалась большая буковка «эн»? В принципе, её можно было дополнительно прилепить в решение, но в целях сократить запись, я этого не сделал. Надеюсь, всем понятно, что и – это обозначение одного и того же.

    Популярный пример для самостоятельного решения:

    Пример 2

    Найти площадь треугольника, построенного на векторах , если

    Формула нахождения площади треугольника через векторное произведение дана в комментариях к определению. Решение и ответ в конце урока.

    На практике задача действительно очень распространена, треугольниками вообще могут замучить.

    Для решения других задач нам понадобятся:

    Свойства векторного произведения векторов

    Некоторые свойства векторного произведения мы уже рассмотрели, тем не менее, я их включу в данный список.

    Для произвольных векторов и произвольного числа справедливы следующие свойства:

    1) В других источниках информации данный пункт обычно не выделяют в свойствах, но он очень важен в практическом плане. Поэтому пусть будет.

    2) – свойство тоже разобрано выше, иногда его называют антикоммутативностью . Иными словами, порядок векторов имеет значение.

    3) – сочетательные или ассоциативные законы векторного произведения. Константы безпроблемно выносятся за пределы векторного произведения. Действительно, чего им там делать?

    4) – распределительные или дистрибутивные законы векторного произведения. С раскрытием скобок тоже нет проблем.

    В качестве демонстрации рассмотрим коротенький пример:

    Пример 3

    Найти , если

    Решение: По условию снова требуется найти длину векторного произведения. Распишем нашу миниатюру:

    (1) Согласно ассоциативным законам, выносим константы за переделы векторного произведения.

    (2) Выносим константу за пределы модуля, при этом модуль «съедает» знак «минус». Длина же не может быть отрицательной.

    (3) Дальнейшее понятно.

    Ответ :

    Пора подбросить дров в огонь:

    Пример 4

    Вычислить площадь треугольника, построенного на векторах , если

    Решение : Площадь треугольника найдём по формуле . Загвоздка состоит в том, что векторы «цэ» и «дэ» сами представлены в виде сумм векторов. Алгоритм здесь стандартен и чем-то напоминает примеры № 3 и 4 урока Скалярное произведение векторов . Решение для ясности разобьём на три этапа:

    1) На первом шаге выразим векторное произведение через векторное произведение , по сути, выразим вектор через вектор . О длинах пока ни слова!

    (1) Подставляем выражения векторов .

    (2) Используя дистрибутивные законы, раскрываем скобки по правилу умножения многочленов.

    (3) Используя ассоциативные законы, выносим все константы за пределы векторных произведений. При маломальском опыте действия 2 и 3 можно выполнять одновременно.

    (4) Первое и последнее слагаемое равно нулю (нулевому вектору) благодаря приятному свойству . Во втором слагаемом используем свойство антикоммутативности векторного произведения:

    (5) Приводим подобные слагаемые.

    В результате вектор оказался выражен через вектор, чего и требовалось достичь:

    2) На втором шаге найдем длину нужного нам векторного произведения. Данное действие напоминает Пример 3:

    3) Найдём площадь искомого треугольника:

    Этапы 2-3 решения можно было оформить и одной строкой.

    Ответ :

    Рассмотренная задача достаточно распространена в контрольных работах, вот пример для самостоятельного решения:

    Пример 5

    Найти , если

    Краткое решение и ответ в конце урока. Посмотрим, насколько вы были внимательны при изучении предыдущих примеров;-)

    Векторное произведение векторов в координатах , заданных в ортонормированном базисе , выражается формулой :

    Формула и правда простецкая: в верхнюю строку определителя записываем координатные векторы, во вторую и третью строки «укладываем» координаты векторов , причём укладываем в строгом порядке – сначала координаты вектора «вэ», затем координаты вектора «дубль-вэ». Если векторы нужно умножить в другом порядке, то и строки следует поменять местами:

    Пример 10

    Проверить, будут ли коллинеарны следующие векторы пространства:
    а)
    б)

    Решение : Проверка основана на одном из утверждений данного урока: если векторы коллинеарны, то их векторное произведение равно нулю (нулевому вектору): .

    а) Найдём векторное произведение:

    Таким образом, векторы не коллинеарны.

    б) Найдём векторное произведение:

    Ответ : а) не коллинеарны, б)

    Вот, пожалуй, и все основные сведения о векторном произведении векторов.

    Данный раздел будет не очень большим, так как задач, где используется смешанное произведение векторов, немного. Фактически всё будет упираться в определение, геометрический смысл и пару рабочих формул.

    Смешанное произведение векторов – это произведение трёх векторов :

    Вот так вот они выстроились паровозиком и ждут, не дождутся, когда их вычислят.

    Сначала опять определение и картинка:

    Определение : Смешанным произведением некомпланарных векторов , взятых в данном порядке , называется объём параллелепипеда , построенного на данных векторах, снабжённый знаком «+», если базис правый, и знаком «–», если базис левый.

    Выполним рисунок. Невидимые нам линии прочерчены пунктиром:

    Погружаемся в определение:

    2) Векторы взяты в определённом порядке , то есть перестановка векторов в произведении , как вы догадываетесь, не проходит без последствий.

    3) Перед тем, как прокомментировать геометрический смысл, отмечу очевидный факт: смешанное произведение векторов является ЧИСЛОМ : . В учебной литературе оформление может быть несколько другим, я привык обозначать смешанное произведение через , а результат вычислений буквой «пэ».

    По определению смешанное произведение – это объем параллелепипеда , построенного на векторах (фигура прочерчена красными векторами и линиями чёрного цвета). То есть, число равно объему данного параллелепипеда.

    Примечание : чертёж является схематическим.

    4) Не будем заново париться с понятием ориентации базиса и пространства. Смысл заключительной части состоит в том, что к объёму может добавляться знак минус. Простыми словами, смешанное произведение может быть отрицательным: .

    Непосредственно из определения следует формула вычисления объема параллелепипеда, построенного на векторах .

    Векторы примеры решения задач, формулы и онлайн калькуляторы

    Содержание:

    Вектора применяются во многих науках, таких как: математика, физика, геометрия и многих других прикладных науках. На практике, они позволяют не делать лишних операций и сократить время выполнения задач. Поэтому, будущим специалистам очень важно понять теорию векторов и научиться решать задачи с ними.

    Перед изучением примеров решения задач советуем изучить теоретический материал по векторам, прочитать все определения и свойства. Список тем находится в правом меню.


    Координаты вектора

    Теоретический материал по теме — координаты вектора.

    Пример

    Запись $\overline{a}=(5 ;-2)$ означает, что вектор $\overline{a}$ имеет следующие координаты: абсцисса равна 5, ордината равна -2.

    Слишком сложно?

    Примеры решения задач с векторами не по зубам? Тебе ответит эксперт через 10 минут!

    Пример

    Задание.{\circ}$$

    Разложение вектора по ортам координатных осей

    Теоретический материал по теме — разложение вектора по ортам.

    Пример

    Задание. Зная разложения вектора $\overline{a}$ по базисной системе векторов: $\overline{a}=3 \overline{i}-\overline{k}$, записать координаты этого вектора в пространстве.

    Решение. Коэффициенты при ортах и есть координатами вектора, поэтому из того, что $\overline{a}=3 \overline{i}-0 \cdot \overline{j}-\overline{k}$, получаем, что $\overline{a}=(3 ; 0 ;-1)$

    Пример

    Задание. Вектор $\overline{a}$ задан своими координатами: $\overline{a}=(2 ;-1 ; 5)$. Записать разложение данного вектора по ортам осей координат.

    Решение. Координаты вектора — это коэффициенты при ортах координатных осей в разложении вектора по базисной системе векторов, поэтому искомое разложение:

    $\overline{a}=2 \overline{i}-\overline{j}+5 \overline{k}$


    Скалярное произведение векторов

    Теоретический материал по теме — скалярное произведение векторов.{\circ}=6 \cdot \frac{1}{2}=3$

    Пример

    Задание. Найти скалярное произведение векторов $\overline{a}=(3 ;-1)$ и $\overline{b}=(-2 ; 7)$

    Решение. Скалярное произведение

    $\overline{a} \overline{b}=3 \cdot(-2)+(-1) \cdot 7=-6-7=-13$


    Векторное произведение векторов

    Теоретический материал по теме — векторное произведение векторов.

    Пример

    Задание. Найти векторное произведение векторов $\overline{a}=(6 ; 7 ; 10)$ и $\overline{b}=(8 ; 5 ; 9)$

    Решение. Составляем определитель и вычисляем его:

    $\overline{a} \times \overline{b}=\left| \begin{array}{ccc}{\overline{i}} & {\overline{j}} & {\overline{k}} \\ {6} & {7} & {10} \\ {8} & {5} & {9}\end{array}\right|=\overline{i} \left| \begin{array}{cc}{7} & {10} \\ {5} & {9}\end{array}\right|-\overline{j} \left| \begin{array}{cc}{6} & {10} \\ {8} & {9}\end{array}\right|+\overline{k} \left| \begin{array}{cc}{6} & {7} \\ {8} & {5}\end{array}\right|=$

    $=\overline{i}(7 \cdot 9-5 \cdot 10)-\overline{j}(6 \cdot 9-8 \cdot 10)+\overline{k}(6 \cdot 5-8 \cdot 7)=$

    $=13 \overline{i}+26 \overline{j}-26 \overline{k}=(13 ; 26 ;-26)$

    Смешанное произведение векторов

    Теоретический материал по теме — смешанное произведение векторов.

    Пример

    Задание. Вычислить объем пирамиды, построенной на векторах $\overline{a}=(2 ; 3 ; 5)$, $\overline{b}=(1 ; 4 ; 4)$, $\overline{c}=(3 ; 5 ; 7)$

    Решение. Найдем смешанное произведение заданных векторов, для это составим определитель, по строкам которого запишем координаты векторов $\overline{a}$, $\overline{b}$ и $\overline{c}$:

    $(\overline{a}, \overline{b}, \overline{c})=\left| \begin{array}{lll}{2} & {3} & {5} \\ {1} & {4} & {4} \\ {3} & {5} & {7}\end{array}\right|=2 \cdot 4 \cdot 7+1 \cdot 5 \cdot 5+3 \cdot 4 \cdot 3-$

    $-3 \cdot 4 \cdot 5-5 \cdot 4 \cdot 2-1 \cdot 3 \cdot 7=-4$

    $$V_{пир}=\frac{1}{6}|(\overline{a}, \overline{b}, \overline{c})|=\frac{1}{6} \cdot 4=\frac{2}{3}$$

    Читать первую тему — операции над векторами, раздела векторы.

    ИСПОЛЬЗОВАНИЕ ВЕКТОРНОГО ПРОИЗВЕДЕНИЯ ВЕКТОРОВ ДЛЯ ВЫЧИСЛЕНИЯ ПЛОЩАДИ НЕКОТОРЫХ ГЕОМЕТРИЧЕСКИХ ФИГУР Исследовательская работа по математике ученика 10 Б класса МОУ СОШ №73 Перевозникова Михаила

    Использование векторного произведения ВЕКТОРОВ

    для вычисления площади

    некоторых геометрических фигур

    Исследовательская работа по математике

    Ученика 10 Б класса

    МОУ СОШ №73

    Перевозникова Михаила

    Руководители:

    Ассистент каф. математического анализа механико-математического факультета СГУ им. Н.Г. Чернышевского Бердников Глеб Сергеевич

    Саратов, 2015

    Содержание

    Введение.

    1. Теоретический обзор.

    1.1. Векторы и вычисления с векторами.

    1.2. Использование скалярного произведения векторов в решении задач

    1.3 Скалярное произведение векторов в координатах

    1.4. Векторное произведение векторов в трёхмерном Евклидовом пространстве: определение понятия.

    1.5. Координаты векторного произведения векторов.

    2. Практическая часть.

    2.1. Связь векторного произведения с площадью треугольника и параллелограмма. Выведение формулы и геометрический смысл векторного произведения векторов.

    2.2. Зная только координаты точек, найти площадь треугольника. Доказательство теоремы

    2.3. Проверка на примерах правильности формулы.

    2.4. Практическое использование векторной алгебры и произведения векторов.

    Заключение

    Введение

    Как известно, многие геометрические задачи имеют два ключевых способа решения – графический и аналитический. Графический метод связан с построением графиков и чертежей, а аналитический    предполагает решение задач преимущественно с помощью алгебраических действий. В последнем случае алгоритм решений задач связан с аналитической геометрией. Аналитическая геометрия – это область математики, а точнее линейной алгебры, которая рассматривает решение геометрических задач средствами алгебры на основе метода координат на плоскости и в пространстве. Аналитическая геометрия позволяет анализировать геометрические образы, исследовать линии и поверхности, важные для практических приложений. При этом в этой науке для расширения пространственного понимания фигур помимо скалярного произведения векторов иногда применяется векторное произведение векторов.

    В связи с широким распространением трехмерных пространственных технологий, изучение свойств некоторых геометрических фигур с использованием векторного произведения представляется актуальным.

    В связи с этим была обозначена цель данного проекта – использование векторного произведения векторов для вычисления площади некоторых геометрических фигур.

    В связи с поставленной целью решались следующие задачи:

    1. Теоретически изучить необходимые основы векторной алгебры и дать определение векторному произведению векторов в системе координат;

    2. Проанализировать наличие связи векторного произведения с площадью треугольника и параллелограмма;

    3. Вывести формулу площади треугольника и параллелограмма в координатах;

    4. Проверить на конкретных примерах верность выведенной формулы.

    1. Теоретический обзор.

      1. Векторы и вычисления с векторами

    Вектором называется направленный отрезок, для которого указано его начало и конец:

    В данном случае началом отрезка является точка А, концом отрезка – точка В. Сам вектор обозначен через или  . Чтобы найти координаты вектора , зная координаты его начальной точек А и конечной точки В, необходимо из координат конечной точки вычесть соответствующие координаты начальной точки:

     = {Bx — Ax ; By — Ay}

    Коллинеарными называются векторы, лежащие на параллельных прямых или на одной прямой. При этом вектор   отрезок, характеризующийся длиной и направлением.

    Длина направленного отрезка определяет числовое значение вектора и называется длиной вектора или модулем вектора.

    Длина вектора || в прямоугольных декартовых координатах равна квадратному корню из суммы квадратов его координат.

    С векторами можно совершать различные действия.

    Например, сложение. Чтобы их сложить, нужно провести сначала второй вектор из конца первого, а потом соединить начало первого с концом второго (рис. 1). Суммой векторов является другой вектор с новыми координатами.

    Сумму векторов   = {ax ; ay} и = {bx ; by} можно найти воспользовавшись следующей формулой:

    = {ax + bx; ay + by}

    Рис. 1. Действия с векторами

    Вычитая векторы, нужно сначала провести их из одной точки, а потом соединить конец второго с концом первого.

    Разность векторов  = {ax ; ay} и  = {bx ; by} можно найти по формуле:

    —  = {ax — bx; ay — by}

    Также, векторы можно умножать на число. Результатом также будет вектор, который в k раз больше (или меньше) данного. Его направление будет зависеть от знака k: при положительном k векторы сонаправлены, а при отрицательном – противоположно направлены.

    Произведение вектора  = {ax ; ay} и числа k можно найти воспользовавшись следующей формулой:

    k ·  = {k · ax; k · ay}

    А можно ли умножать вектор на вектор? Конечно, и даже двумя вариантами!

    Первый вариант – скалярное произведение.

    Рис. 2. Скалярное произведение в координатах

    Для нахождения произведения векторов можно использовать угол  между данными векторами, показанный на рисунке 3.

    Из формулы следует, что скалярное произведение равно произведению длин данных векторов на косинус угла между ними, его результатом является число. Важно, что если векторы перпендикулярны, то их скалярное произведение равно нулю, т.к. косинус прямого угла между ними равен нулю.

    В координатной плоскости вектор также имеет координаты. Вектора, их координаты и скалярное произведение являются одними из самых удобных методов вычисления угла между прямыми (или их отрезками), если введена система координат. И если координаты , то их скалярное произведение равно:

    В трехмерном пространстве существует 3 оси и, соответственно, у точек и векторов в такой системе будет по 3 координаты, а скалярное произведение векторов вычисляется по формуле:

    .

    1.2. Векторное произведение векторов в трехмерном пространстве.

    Вторым вариантом вычисления произведения векторов является векторное произведение. Но, чтобы его определить требуется уже не плоскость, а трехмерное пространство, в котором начало и конец вектора имеют по 3 координаты.

    В отличие от скалярного произведения векторов в трёхмерном пространстве операция «векторное умножение» над векторами приводит к иному результату. Если в предыдущем случае скалярного умножения двух векторов результатом было число, то в случае векторного умножения векторов результатом будет другой вектор, перпендикулярный обоим вступившим в произведение векторам. Поэтому это произведение векторов называется векторным.

    Очевидно, что при построении результирующего вектора , перпендикулярного двум, вступившим в произведение — и , может быть выбрано два противоположных направления. При этом направление результирующего вектора определяется по правилу правой руки, или правилу буравчика. Если нарисовать векторы так, чтобы их начала совпадали и вращать первый вектор-сомножитель кратчайшим образом ко второму вектору-сомножителю, а четыре пальца правой руки показывали направление вращения (как бы охватывая вращающийся цилиндр), то оттопыренный большой палец покажет направление вектора-произведения (рис. 7).

    Рис. 7. Правило правой руки

    1.3. Свойства векторного произведения векторов.

    Длина результирующего вектора определяется по формуле

    .

    При этом   векторное произведение. Как было сказано выше, результирующий вектор будет перпендикулярен , а его направление определяется по правилу правой руки.

    Векторное произведение зависит от порядка сомножителей, именно:

    .

    Векторное произведение ненулевых векторов равно 0, если они коллинеарны, тогда синус угла между ними будет равен 0.

    Координаты векторов в трехмерном пространстве выражаются следующим образом: . Тогда координаты результирующего вектора находим по формуле

    .

    Длина результирующего вектора находится по формуле:

    .



    2. Практическая часть.

    2.1. Связь векторного произведения с площадью треугольника и параллелограмма в плоскости. Геометрический смысл векторного произведения векторов.

    Пусть нам дан треугольник ABC (рис. 8). Известно, что .

    Если представить стороны треугольника АВ и АС в виде двух векторов, то в формуле площади треугольника мы находим выражение векторного произведения векторов:

    Из выше сказанного можно определить геометрический смысл векторного произведения (рис. 9):

    длина векторного произведения векторов равна удвоенной площади треугольника, имеющего сторонами векторы  и , если их отложить от одной точки.

    Другими словами, длина векторного произведения векторов  и  равна площади параллелограмма, построенного на векторах  и , со сторонами  и  и углом между ними, равным .

    .

    Рис. 9. Геометрический смысл векторного произведения векторов

    В связи с этим, можно привести еще одно определение векторного произведения векторов:

    Векторным произведением вектора  на вектор   называется вектор , длина которого численно равна площади параллелограмма построенного на векторах  и , перпендикулярный к плоскости этих векторов и направленный так, чтоб наименьшее вращение от  к  вокруг вектора  осуществлялось против часовой стрелки, если смотреть с конца вектора  (рис. 10).


    Рис. 10. Определение векторного произведения векторов

    с использованием параллелограмма

    2.2. Вывод формулы для нахождения площади треугольника в координатах.

    Итак, нам дан треугольник АВС в плоскости и координаты его вершин. Найдем площадь этого треугольника (рис. 11).

    Рис. 11. Пример решения задачи на нахождение площади треугольника по координатам его вершин

    Решение.

    Для начала, рассмотрим координаты вершин в пространстве и вычислим координаты векторов АВ и АС.

    По данной прежде формуле подсчитаем координаты их векторного произведения. Длина этого вектора равна 2 площадям треугольника АВС. Площадь треугольника равна 10.

    Более того, если мы рассмотрим треугольник на плоскости, то первые 2 координаты векторного произведения всегда будут равны нулю, поэтому мы можем сформулировать следующую теорему.

    Теорема: Пусть дан треугольник АВС и координаты его вершин (рис. 12).

    Тогда .

    Рис. 12. Доказательство теоремы

    Доказательство.

    Рассмотрим точки в пространстве и вычислим координаты векторов ВС и ВА. . По приведенной раньше формуле вычислим координаты векторного произведения этих векторов. Обратим внимание, что все члены, содержащие z1 или z2, равны 0, т.к. z1и z2 = 0. УБРАТЬ!!!

    Итак, следовательно,

    2.3. Проверка правильности формулы на примерах

    Найти площадь треугольника образованного векторами a = {-1; 2; -2} и b = {2; 1; -1}.

    Решение: Найдем векторное произведение этих векторов:

    a × b= 

     i 

     j 

     k 

     =

     -1 

     2 

     -2 

     2 

     1 

     -1 

    = i(2 · (-1) — (-2) · 1) — j((-1) · (-1) — (-2) · 2) + k((-1) · 1 — 2 · 2) =

    = i(-2 + 2) — j(1 + 4) + k(-1 — 4) = -5j — 5k = {0; -5; -5}

    Из свойств векторного произведения:

    SΔ = 

    1

    |a × b| = 

    1

    √02 + 52 + 52 = 

    1

    √25 + 25 = 

    1

    √50 = 

    5√2

    2

    2

    2

    2

    2

    Ответ: SΔ = 2.5√2.

    Заключение

    2.4. Приложения векторной алгебры

    и скалярного и векторного произведения векторов.

    Где же нужны векторы? Векторное пространство и векторы носят не только теоретический характер, но и имеют вполне реальное практическое применение в современном мире.

    В механике и физике многие величины имеют не только численное значение, но и направление. Такие величины называются векторными. Вместе с использованием элементарных механических понятий, опираясь на их физический смысл, многие величины рассматриваются как скользящие векторы, а их свойства описываются как аксиомами, как это принято в теоретической механике, так и при помощи математических свойств векторов. Наиболее яркими примерами векторных величин являются скорость, импульс и сила (рис. 12). Например, момент импульса и сила Лоренца математически записываются с помощью векторов.

    В физике важны не только сами вектора, но в большой степени важны и их произведения, которые помогают вычислять некоторые величины. Векторное произведение полезно для определения коллинеарности векторов   модуль векторного произведения двух векторов равен произведению их модулей, если они перпендикулярны, и уменьшается до нуля, если векторы сонаправленны или противоположно направленны.

    Еще один пример: скалярное произведение используется для вычисления работы по приведенной ниже формуле, где F – вектор силы, а s – вектор перемещения.

    Одним из примеров использования произведения векторов является момент силы, равный произведению радиус-вектора, проведенного от оси вращения к точке приложения силы, на вектор этой силы.

    Многое из того, что вычисляется в физике по правилу правой руки является векторным произведением. Найти подтверждения, привести примеры.

    Стоит еще заметить, что двухмерным и трехмерным пространством не исчерпываются возможные варианты векторных пространств. Высшая математика рассматривает пространства большей размерности, в которых также определяются аналоги формул для скалярного и векторного произведения. Несмотря на то, что пространства большей размерности, чем 3, человеческое сознание неспособно представить визуально, они удивительным образом находят себе приложения во многих областях науки и промышленности.

    В то же время результатом векторного произведения векторов в трёхмерном Евклидовом пространстве является не число, а результирующий вектор со своими координатами, направлением и длиной.

    Направление результирующего вектора определяется по правилу правой руки, что является одним из самых удивительных положений аналитической геометрии.

    Векторное произведение векторов может быть использовано в нахождении площади треугольника или параллелограмма по заданным координатам вершин, что было подтверждено выведением формулы, доказательством теоремы и решением практических задач.

    Векторы широко используются в физике, где такие показатели как скорость, импульс и сила могут быть представлены в виде векторных величин и вычисляются геометрически.

    Список использованных источников

    Атанасян Л. С., Бутузов В. Ф., Кадомцев С. Б. и др. Геометрия. 7-9 классы: учебник для общеобразовательных организаций. М.: Просвещение, 2013. 383 с.

    Атанасян Л.С., Бутузов В. Ф., Кадомцев С. Б. и др. Геометрия. 10-11 классы: учебник для общеобразовательных организаций: базовый и профильный уровни. М.: Просвещение, 2013. 255 с.

    Бугров Я.С., Никольский С.М. Высшая математика. Том первый: элементы линейной алгебры и аналитической геометрии.

    ????Клетеник Д.В. Сборник задач по аналитической геометрии. М.: Наука, Физматлит, 1998.

    Аналитическая геометрия.

    http://a-geometry.narod.ru/problems/problems_32.htm

    Математика. Клевер.

    http://www.cleverstudents.ru/vectors/vector_product_of_vectors.html

    ——Изучение математики онлайн.

    http://ru.onlinemschool.com/math/library/vector/multiply1/

    Сайт В. Глазнева.

    http://glaznev.sibcity.ru/1kurs/analit/common/html/anlek7.htm

    ——Википедия.

    https://ru.wikipedia.org/wiki/%C2%E5%EA%F2%EE%F0%ED%EE%E5_%EF%F0%EE%E8%E7%E2%E5%E4%E5%ED%E8%E5

    Найти смешанное произведение векторов онлайн калькулятор. Смешанное произведение векторов

    Для того, чтобы подробно рассмотреть такую тему, нужно охватить еще несколько разделов. Тема напрямую связана с такими терминами, как скалярное и векторное произведение. В этой статье мы постарались дать точное определение, указать формулу, которая поможет определить произведение, используя координаты векторов. Помимо этого, статья включает в себя разделы с перечислением свойств произведения и представлены подробный разбор типовых равенств и задач.

    Yandex.RTB R-A-339285-1

    Термин

    Для того, чтобы определить, в чем заключается данный термин, нужно взять три вектора.

    Определение 1

    Смешанным произведением a → , b → и d → является та величина, которая равняется скалярному произведению a → × b → и d → , где a → × b → — умножение a → и b → . Операцию умножения a → , b → и d → зачастую обозначают a → · b → · d → . Можно преобразовать формулу так: a → · b → · d → = (a → × b → , d →) .

    Умножение в системе координат

    Мы можем умножить вектора, если они указаны на координатной плоскости.

    Возьмем i → , j → , k →

    Произведение векторов в данном конкретном случае будет иметь следующий вид: a → × b → = (a y · b z — a z · b y) · i → + (a z · b x + a x · b z) · j → + (a x · b y + a y · b x) · k → = a y a z b y b z · i → — a x a z b x b z · j → + a x a y b x b y · k →

    Определение 2

    Для выполнения скалярного произведения в системе координат необходимо сложить результаты, полученный во время умножения координат.

    Из этого следует:

    a → × b → = (a y · b z — a z · b y) · i → + (a z · b x + a x · b z) · j → + (a x · b y + a y · b x) · k → = a y a z b y b z · i → — a x a z b x b z · j → + a x a y b x b y · k →

    Мы также можем определить смешанное произведение векторов, если в заданной системе координат указаны координаты векторов, которые умножаются.

    a → × b → = (a y a z b y b z · i → — a x a z b x b z · j → + a x a y b x b y · k → , d x · i → + d y · j → + d z · k →) = = a y a z b y b z · d x — a x a z b x b z · d y + a x a y b x b y · d z = a x a y a z b x b y b z d x d y d z

    Таким образом, можно сделать вывод, что:

    a → · b → · d = a → × b → , d → = a x a y a z b x b y b z d x d y d z

    Определение 3

    Смешанное произведение можно приравнять к определителю матрицы, в качестве строк которой использованы векторные координаты. Наглядно это выглядит так: a → · b → · d = a → × b → , d → = a x a y a z b x b y b z d x d y d z .

    Свойства операции над векторами Из особенностей, которые выделяются в скалярном или векторном произведении, можно вывести особенности, которые характеризуют смешанное произведение. Ниже мы приведем основные свойства.

    1. (λ · a →) · b → · d → = a → · (λ · b →) · d → = a → · b → · (λ · d →) = λ · a → · b → · d → λ ∈ R ;
    2. a → · b → · d → = d → · a → · b → = b → · d → · a → ; a → · d → · b → = b → · a → · d → = d → · b → · a → ;
    3. (a (1) → + a (2) →) · b → · d → = a (1) → · b → · d → + a (2) → · b → · d → a → · (b (1) → + b (2) →) · d → = a → · b (1) → · d → + a → · b (2) → · d → a → · b → · (d (1) → + d (2) →) = a → · b → · d (2) → + a → · b → · d (2) →

    Помимо приведенных свойств, следует уточнить, что если множитель нулевой, то результатом умножения также станет нуль.

    Результатом умножения также будет нуль в том случае, если два или больше множителей равны.

    Действительно, если a → = b → , то, следуя определению векторного произведения [ a → × b → ] = a → · b → · sin 0 = 0 , следовательно, смешанное произведение равно нулю, так как ([ a → × b → ] , d →) = (0 → , d →) = 0 .

    Если же a → = b → или b → = d → , то угол между векторами [ a → × b → ] и d → равен π 2 . По определению скалярного произведения векторов ([ a → × b → ] , d →) = [ a → × b → ] · d → · cos π 2 = 0 .

    Свойства операции умножения чаще всего требуются во время решения задач.
    Для того, чтобы подробно разобрать данную тему, возьмем несколько примеров и подробно их распишем.

    Пример 1

    Докажите равенство ([ a → × b → ] , d → + λ · a → + b →) = ([ a → × b → ] , d →) , где λ — некоторое действительное число.

    Для того, чтобы найти решение этого равенства, следует преобразовать его левую часть. Для этого необходимо воспользоваться третьим свойством смешанного произведения, которое гласит:

    ([ a → × b → ] , d → + λ · a → + b →) = ([ a → × b → ] , d →) + ([ a → × b → ] , λ · a →) + ([ a → × b → ] , b →)
    Мы разобрали, что (([ a → × b → ] , b →) = 0 . , d →) ≤ ≤ a → · b → · 1 · d → · 1 = a → · b → · d →

    Неравенство доказано.

    Разбор типовых задач

    Для того, чтобы определить, чему равно произведение векторов, следует знать координаты умножаемых векторов. Для операции можно использовать такую формулу a → · b → · d → = (a → × b → , d →) = a x a y a z b x b y b z d x d y d z .

    Пример 3

    В прямоугольной системе координат представлены 3 вектора с такими координатами: a → = (1 , — 2 , 3) , b → (- 2 , 2 , 1) , d → = (3 , — 2 , 5) . Необходимо определить, чему равно произведение указанных векторов a → · b → · d → .

    Исходя из теории, представленной выше, мы можем воспользоваться правилом, которое гласит, что смешанное произведение может быть вычислено через определитель матрицы. Это будет выглядеть так: a → · b → · d → = (a → × b → , d →) = a x a y a z b x b y b z d x d y d z = 1 — 2 3 — 2 2 1 3 — 2 5 = = 1 · 2 · 5 + (- 1) · 1 · 3 + 3 · (- 2) · (- 2) — 3 · 2 · 3 — (- 1) · (- 2) · 5 — 1 · 1 · (- 2) = — 7

    Пример 4

    Необходимо найти произведение векторов i → + j → , i → + j → — k → , i → + j → + 2 · k → , где i → , j → , k → — орты прямоугольной декартовой системы координат.

    Исходя из условия, которое гласит, что вектора расположены в данной системе координат, можно вывести их координаты: i → + j → = (1 , 1 , 0) i → + j → — k → = (1 , 1 , — 1) i → + j → + 2 · k → = (1 , 1 , 2)

    Используем формулу, которая использовалась выше
    i → + j → × (i → + j → — k → , (i → + j → + 2 · k →) = 1 1 0 1 1 — 1 1 1 2 = 0 i → + j → × (i → + j → — k → , (i → + j → + 2 · k →) = 0

    Смешанное произведение также возможно определить с помощью длины вектора, которая уже известна, и угла между ними. Разберем этот тезис в примере.

    Пример 5

    В прямоугольной системе координат расположены три вектора a → , b → и d → , которые перпендикулярны между собой. Они представляют собой правую тройку, их длины составляют 4 , 2 и 3 . Необходимо умножить вектора.

    Обозначим c → = a → × b → .

    Согласно правилу, результатом умножения скалярных векторов является число, которое равно результату умножения длин используемых векторов на косинус угла между ними.) = c → · n p c → d → , где n p c → d → — числовая проекция вектора d → на направление вектора c → = [ a → × b → ] .

    Абсолютная величина n p c → d → равняется числу, которое также является равно высоте фигуры, для которого использованы вектора a → , b → и d → в качестве сторон. Исходя из этого, следует уточнить, что c → = [ a → × b → ] перпендикулярен a → и вектору и вектору согласно определению умножения векторов. Величина c → = a → x b → равняется площади параллелепипеда, построенного на векторах a → и b → .

    Делаем вывод, что модуль произведения a → · b → · d → = c → · n p c → d → равен результату умножения площади основания на высоту фигуры, которая построена на векторах a → , b → и d → .

    Определение 4

    Абсолютная величина векторного произведения является объемом параллелепипеда : V п а р а л л е л е п и п и д а = a → · b → · d → .

    Данная формула и является геометрическим смыслом.

    Определение 5

    Объем тетраэдра , который построен на a → , b → и d → , равняется 1 / 6 объема параллелепипеда Получаем, V т э т р а э д а = 1 6 · V п а р а л л е л е п и п и д а = 1 6 · a → · b → · d → .

    Для того, чтобы закрепить знания, разберем несколько типичных примеров

    Пример 6

    Необходимо найти объем параллелепипеда, в качестве сторон которого используются A B → = (3 , 6 , 3) , A C → = (1 , 3 , — 2) , A A 1 → = (2 , 2 , 2) , заданные в прямоугольной системе координат. Объем параллелепипеда можно найти, используя формулу об абсолютной величине. Из этого следует: A B → · A C → · A A 1 → = 3 6 3 1 3 — 2 2 2 2 = 3 · 3 · 2 + 6 · (- 2) · 2 + 3 · 1 · 2 — 3 · 3 · 2 — 6 · 1 · 2 — 3 · (- 2) · 2 = — 18

    Тогда, V п а р а л л е л е п и п е д а = — 18 = 18 .

    V п а р а л л е л е п и п и д а = 18

    Пример 7

    В системе координат заданы точки A (0 , 1 , 0) , B (3 , — 1 , 5) , C (1 , 0 , 3) , D (- 2 , 3 , 1) . Следует определить объем тетраэдра, который расположен на этих точках.

    Воспользуемся формулой V т э т р а э д р а = 1 6 · A B → · A C → · A D → . Мы можем определить координаты векторов по координатам точек: A B → = (3 — 0 , — 1 — 1 , 5 — 0) = (3 , — 2 , 5) A C → = (1 — 0 , 0 — 1 , 3 — 0) = (1 , — 1 , 3) A D → = (- 2 — 0 , 3 — 1 , 1 — 0) = (- 2 , 2 , 1)

    Дальше определяем смешанное произведение A B → · A C → · A D → по координатам векторов: A B → · A C → · A D → = 3 — 2 5 1 — 1 3 — 2 2 1 = 3 · (- 1) · 1 + (- 2) · 3 · (- 2) + 5 · 1 · 2 — 5 · (- 1) · (- 2) — (- 2) · 1 · 1 — 3 · 3 · 2 = — 7 Объем V т э т р а э д р а = 1 6 · — 7 = 7 6 .

    V т э т р а э д р а = 7 6 .

    Если вы заметили ошибку в тексте, пожалуйста, выделите её и нажмите Ctrl+Enter

    Смешанным (или векторно-скалярным) произведением трех векторов a, b, c (взятых в указанном порядке) называется скалярное произведение вектора a на векторное произведение b x c , т. е. число a(b x c), или, что то же, (b x c)a.
    Обозначение: abc .

    Назначение . Онлайн-калькулятор предназначен для вычисления смешанного произведения векторов. Полученное решение сохраняется в файле Word . Дополнительно создается шаблон решения в Excel .

    Признаки компланарности векторов

    Три вектора (или большее число) называются компланарными, если они, будучи приведены к общему началу, лежат в одной плоскости.
    Если хотя бы один из трех векторов – нулевой, то три вектора тоже считаются компланарными.

    Признак компланарности . Если система a, b, c – правая, то abc>0 ; если левая, то abcГеометрический смысл смешанного произведения . Смешанное произведение abc трех некомпланарных векторов a, b, c равно объему параллелепипеда, построенного на векторах a, b, c , взятому со знаком плюс, если система a, b, c – правая, и со знаком минус, если эта система левая.

    Свойства смешанного произведения

    1. При круговой перестановке сомножителей смешанное произведение не меняется, при перестановке двух сомножителей – меняет знак на обратный: abc=bca=cab=-(bac)=-(cba)=-(acb)
      Вытекает из геометрического смысла.
    2. (a+b)cd=acd+bcd (распределительное свойство). Распространяется на любое число слагаемых.
      Вытекает из определения смешанного произведения.
    3. (ma)bc=m(abc) (сочетательное свойство относительно скалярного множителя).
      Вытекает из определения смешанного произведения. Эти свойства позволяют применять к смешанным произведениям преобразования, отличающиеся от обычных алгебраических лишь тем, что менять порядок сомножителей можно только с учетом знака произведения.
    4. Смешанное произведение, имеющее хотя бы два равных сомножителя, равно нулю: aab=0 .

    Пример №1 . Найти смешанное произведение. ab(3a+2b-5c)=3aba+2abb-5abc=-5abc .

    Пример №2 . (a+b)(b+c)(c+a)= (axb+axc+bxb+bxc)(c+a)= (axb+axc +bxc)(c+a)=abc+acc+aca+aba+bcc+bca . Все члены, кроме двух крайних, равны нулю. Кроме того, bca=abc . Поэтому (a+b)(b+c)(c+a)=2abc .

    Пример №3 . Вычислить смешанное произведение трех векторов a=15i+20j+5k, b=2i-4j+14k, c=3i-6j+21k .
    Решение . Чтобы вычислить смешанное произведение векторов, необходимо найти определитель системы, составленной из координат векторов. Запишем систему в виде.

    На данном уроке мы рассмотрим ещё две операции с векторами: векторное произведение векторов и смешанное произведение векторов (сразу ссылка, кому нужно именно оно) . Ничего страшного, так иногда бывает, что для полного счастья, помимо скалярного произведения векторов , требуется ещё и ещё. Такая вот векторная наркомания. Может сложиться впечатление, что мы залезаем в дебри аналитической геометрии. Это не так. В данном разделе высшей математики вообще мало дров, разве что на Буратино хватит. На самом деле материал очень распространенный и простой – вряд ли сложнее, чем то же скалярное произведение , даже типовых задач поменьше будет. Главное в аналитической геометрии, как многие убедятся или уже убедились, НЕ ОШИБАТЬСЯ В ВЫЧИСЛЕНИЯХ. Повторяйте как заклинание, и будет вам счастье =)

    Если векторы сверкают где-то далеко, как молнии на горизонте, не беда, начните с урока Векторы для чайников , чтобы восстановить или вновь приобрести базовые знания о векторах. Более подготовленные читатели могут знакомиться с информацией выборочно, я постарался собрать максимально полную коллекцию примеров, которые часто встречаются в практических работах

    Чем вас сразу порадовать? Когда я был маленьким, то умел жонглировать двумя и даже тремя шариками. Ловко получалось. Сейчас жонглировать не придётся вообще, поскольку мы будем рассматривать только пространственные векторы , а плоские векторы с двумя координатами останутся за бортом. Почему? Такими уж родились данные действия – векторное и смешанное произведение векторов определены и работают в трёхмерном пространстве. Уже проще!

    В данной операции, точно так же, как и в скалярном произведении, участвуют два вектора . Пусть это будут нетленные буквы .

    Само действие обозначается следующим образом: . Существуют и другие варианты, но я привык обозначать векторное произведение векторов именно так, в квадратных скобках с крестиком.

    И сразу вопрос : если в скалярном произведении векторов участвуют два вектора, и здесь тоже умножаются два вектора, тогда в чём разница ? Явная разница, прежде всего, в РЕЗУЛЬТАТЕ:

    Результатом скалярного произведения векторов является ЧИСЛО:

    Результатом векторного произведения векторов является ВЕКТОР : , то есть умножаем векторы и получаем снова вектор. Закрытый клуб. Собственно, отсюда и название операции. В различной учебной литературе обозначения тоже могут варьироваться, я буду использовать букву .

    Определение векторного произведения

    Сначала будет определение с картинкой, затем комментарии.

    Определение : Векторным произведением неколлинеарных векторов , взятых в данном порядке , называется ВЕКТОР , длина которого численно равна площади параллелограмма , построенного на данных векторах; вектор ортогонален векторам , и направлен так, что базис имеет правую ориентацию:

    Разбираем определение по косточкам, тут много интересного!

    Итак, можно выделить следующие существенные моменты:

    1) Исходные векторы , обозначенные красными стрелками, по определению не коллинеарны . Случай коллинеарных векторов будет уместно рассмотреть чуть позже.

    2) Векторы взяты в строго определённом порядке : – «а» умножается на «бэ» , а не «бэ» на «а». Результатом умножения векторов является ВЕКТОР , который обозначен синим цветом. Если векторы умножить в обратном порядке, то получим равный по длине и противоположный по направлению вектор (малиновый цвет). То есть, справедливо равенство .

    3) Теперь познакомимся с геометрическим смыслом векторного произведения. Это очень важный пункт! ДЛИНА синего вектора (а, значит, и малинового вектора ) численно равна ПЛОЩАДИ параллелограмма, построенного на векторах . На рисунке данный параллелограмм заштрихован чёрным цветом.

    Примечание : чертёж является схематическим, и, естественно, номинальная длина векторного произведения не равна площади параллелограмма.

    Вспоминаем одну из геометрических формул: площадь параллелограмма равна произведению смежных сторон на синус угла между ними . Поэтому, исходя из вышесказанного, справедлива формула вычисления ДЛИНЫ векторного произведения:

    Подчёркиваю, что в формуле речь идёт о ДЛИНЕ вектора, а не о самом векторе . Каков практический смысл? А смысл таков, что в задачах аналитической геометрии площадь параллелограмма часто находят через понятие векторного произведения:

    Получим вторую важную формулу. Диагональ параллелограмма (красный пунктир) делит его на два равных треугольника. Следовательно, площадь треугольника, построенного на векторах (красная штриховка), можно найти по формуле:

    4) Не менее важный факт состоит в том, что вектор ортогонален векторам , то есть . Разумеется, противоположно направленный вектор (малиновая стрелка) тоже ортогонален исходным векторам .

    5) Вектор направлен так, что базис имеет правую ориентацию. На уроке о переходе к новому базису я достаточно подробно рассказал об ориентации плоскости , и сейчас мы разберёмся, что такое ориентация пространства. Объяснять буду на пальцах вашей правой руки . Мысленно совместите указательный палец с вектором и средний палец с вектором . Безымянный палец и мизинец прижмите к ладони. В результате большой палец – векторное произведение будет смотреть вверх. Это и есть правоориентированный базис (на рисунке именно он). Теперь поменяйте векторы (указательный и средний пальцы ) местами, в результате большой палец развернётся, и векторное произведение уже будет смотреть вниз. Это тоже правоориентированный базис. Возможно, у вас возник вопрос: а какой базис имеет левую ориентацию? «Присвойте» тем же пальцам левой руки векторы , и полУчите левый базис и левую ориентацию пространства (в этом случае большой палец расположится по направлению нижнего вектора) . Образно говоря, данные базисы «закручивают» или ориентируют пространство в разные стороны. И это понятие не следует считать чем-то надуманным или абстрактным – так, например, ориентацию пространства меняет самое обычное зеркало, и если «вытащить отражённый объект из зазеркалья», то его в общем случае не удастся совместить с «оригиналом». Кстати, поднесите к зеркалу три пальца и проанализируйте отражение;-)

    …как всё-таки хорошо, что вы теперь знаете о право- и левоориентированных базисах, ибо страшнЫ высказывания некоторых лекторов о смене ориентации =)

    Векторное произведение коллинеарных векторов

    Определение подробно разобрано, осталось выяснить, что происходит, когда векторы коллинеарны. Если векторы коллинеарны, то их можно расположить на одной прямой и наш параллелограмм тоже «складывается» в одну прямую. Площадь такого, как говорят математики, вырожденного параллелограмма равна нулю. Это же следует и из формулы – синус нуля или 180-ти градусов равен нулю, а значит, и площадь нулевая

    Таким образом, если , то . Строго говоря, само векторное произведение равно нулевому вектору, но на практике этим часто пренебрегают и пишут, что оно просто равно нулю.

    Частный случай – векторное произведение вектора на самого себя:

    С помощью векторного произведения можно проверять коллинеарность трёхмерных векторов, и данную задачу среди прочих мы тоже разберём.

    Для решения практических примеров может потребоваться тригонометрическая таблица , чтобы находить по ней значения синусов.

    Ну что же, разжигаем огонь:

    Пример 1

    а) Найти длину векторного произведения векторов , если

    б) Найти площадь параллелограмма, построенного на векторах , если

    Решение : Нет, это не опечатка, исходные данные в пунктах условия я намеренно сделал одинаковыми. Потому что оформление решений будет отличаться!

    а) По условию требуется найти длину вектора (векторного произведения). По соответствующей формуле:

    Ответ :

    Коль скоро спрашивалось о длине, то в ответе указываем размерность – единицы.

    б) По условию требуется найти площадь параллелограмма, построенного на векторах . Площадь данного параллелограмма численно равна длине векторного произведения:

    Ответ :

    Обратите внимание, что в ответе о векторном произведении речи не идёт вообще, нас спрашивали о площади фигуры , соответственно, размерность – квадратные единицы.

    Всегда смотрим, ЧТО требуется найти по условию, и, исходя из этого, формулируем чёткий ответ. Может показаться буквоедством, но буквоедов среди преподавателей хватает, и задание с хорошими шансами вернётся на доработку. Хотя это не особо натянутая придирка – если ответ некорректен, то складывается впечатление, что человек не разбирается в простых вещах и/или не вник в суть задания. Этот момент всегда нужно держать на контроле, решая любую задачу по высшей математике, да и по другим предметам тоже.

    Куда подевалась большая буковка «эн»? В принципе, её можно было дополнительно прилепить в решение, но в целях сократить запись, я этого не сделал. Надеюсь, всем понятно, что и – это обозначение одного и того же.

    Популярный пример для самостоятельного решения:

    Пример 2

    Найти площадь треугольника, построенного на векторах , если

    Формула нахождения площади треугольника через векторное произведение дана в комментариях к определению. Решение и ответ в конце урока.

    На практике задача действительно очень распространена, треугольниками вообще могут замучить.

    Для решения других задач нам понадобятся:

    Свойства векторного произведения векторов

    Некоторые свойства векторного произведения мы уже рассмотрели, тем не менее, я их включу в данный список.

    Для произвольных векторов и произвольного числа справедливы следующие свойства:

    1) В других источниках информации данный пункт обычно не выделяют в свойствах, но он очень важен в практическом плане. Поэтому пусть будет.

    2) – свойство тоже разобрано выше, иногда его называют антикоммутативностью . Иными словами, порядок векторов имеет значение.

    3) – сочетательные или ассоциативные законы векторного произведения. Константы безпроблемно выносятся за пределы векторного произведения. Действительно, чего им там делать?

    4) – распределительные или дистрибутивные законы векторного произведения. С раскрытием скобок тоже нет проблем.

    В качестве демонстрации рассмотрим коротенький пример:

    Пример 3

    Найти , если

    Решение: По условию снова требуется найти длину векторного произведения. Распишем нашу миниатюру:

    (1) Согласно ассоциативным законам, выносим константы за переделы векторного произведения.

    (2) Выносим константу за пределы модуля, при этом модуль «съедает» знак «минус». Длина же не может быть отрицательной.

    (3) Дальнейшее понятно.

    Ответ :

    Пора подбросить дров в огонь:

    Пример 4

    Вычислить площадь треугольника, построенного на векторах , если

    Решение : Площадь треугольника найдём по формуле . Загвоздка состоит в том, что векторы «цэ» и «дэ» сами представлены в виде сумм векторов. Алгоритм здесь стандартен и чем-то напоминает примеры № 3 и 4 урока Скалярное произведение векторов . Решение для ясности разобьём на три этапа:

    1) На первом шаге выразим векторное произведение через векторное произведение , по сути, выразим вектор через вектор . О длинах пока ни слова!

    (1) Подставляем выражения векторов .

    (2) Используя дистрибутивные законы, раскрываем скобки по правилу умножения многочленов.

    (3) Используя ассоциативные законы, выносим все константы за пределы векторных произведений. При маломальском опыте действия 2 и 3 можно выполнять одновременно.

    (4) Первое и последнее слагаемое равно нулю (нулевому вектору) благодаря приятному свойству . Во втором слагаемом используем свойство антикоммутативности векторного произведения:

    (5) Приводим подобные слагаемые.

    В результате вектор оказался выражен через вектор, чего и требовалось достичь:

    2) На втором шаге найдем длину нужного нам векторного произведения. Данное действие напоминает Пример 3:

    3) Найдём площадь искомого треугольника:

    Этапы 2-3 решения можно было оформить и одной строкой.

    Ответ :

    Рассмотренная задача достаточно распространена в контрольных работах, вот пример для самостоятельного решения:

    Пример 5

    Найти , если

    Краткое решение и ответ в конце урока. Посмотрим, насколько вы были внимательны при изучении предыдущих примеров;-)

    Векторное произведение векторов в координатах , заданных в ортонормированном базисе , выражается формулой :

    Формула и правда простецкая: в верхнюю строку определителя записываем координатные векторы, во вторую и третью строки «укладываем» координаты векторов , причём укладываем в строгом порядке – сначала координаты вектора «вэ», затем координаты вектора «дубль-вэ». Если векторы нужно умножить в другом порядке, то и строки следует поменять местами:

    Пример 10

    Проверить, будут ли коллинеарны следующие векторы пространства:
    а)
    б)

    Решение : Проверка основана на одном из утверждений данного урока: если векторы коллинеарны, то их векторное произведение равно нулю (нулевому вектору): .

    а) Найдём векторное произведение:

    Таким образом, векторы не коллинеарны.

    б) Найдём векторное произведение:

    Ответ : а) не коллинеарны, б)

    Вот, пожалуй, и все основные сведения о векторном произведении векторов.

    Данный раздел будет не очень большим, так как задач, где используется смешанное произведение векторов, немного. Фактически всё будет упираться в определение, геометрический смысл и пару рабочих формул.

    Смешанное произведение векторов – это произведение трёх векторов :

    Вот так вот они выстроились паровозиком и ждут, не дождутся, когда их вычислят.

    Сначала опять определение и картинка:

    Определение : Смешанным произведением некомпланарных векторов , взятых в данном порядке , называется объём параллелепипеда , построенного на данных векторах, снабжённый знаком «+», если базис правый, и знаком «–», если базис левый.

    Выполним рисунок. Невидимые нам линии прочерчены пунктиром:

    Погружаемся в определение:

    2) Векторы взяты в определённом порядке , то есть перестановка векторов в произведении , как вы догадываетесь, не проходит без последствий.

    3) Перед тем, как прокомментировать геометрический смысл, отмечу очевидный факт: смешанное произведение векторов является ЧИСЛОМ : . В учебной литературе оформление может быть несколько другим, я привык обозначать смешанное произведение через , а результат вычислений буквой «пэ».

    По определению смешанное произведение – это объем параллелепипеда , построенного на векторах (фигура прочерчена красными векторами и линиями чёрного цвета). То есть, число равно объему данного параллелепипеда.

    Примечание : чертёж является схематическим.

    4) Не будем заново париться с понятием ориентации базиса и пространства. Смысл заключительной части состоит в том, что к объёму может добавляться знак минус. Простыми словами, смешанное произведение может быть отрицательным: .

    Непосредственно из определения следует формула вычисления объема параллелепипеда, построенного на векторах .

    Данный онлайн калькулятор вычисляет смешанное произведение векторов. Дается подробное решение. Для вычисления смешанного произведения векторов выберите способ представления векторов (по координатам или по двум точкам) введите данные в ячейки и нажимайте на кнопку «Вычислить.»

    ×

    Предупреждение

    Очистить все ячейки?

    Закрыть Очистить

    Инструкция ввода данных. Числа вводятся в виде целых чисел (примеры: 487, 5, -7623 и т.д.), десятичных чисел (напр. 67., 102.54 и т.д.) или дробей. Дробь нужно набирать в виде a/b, где a и b (b>0) целые или десятичные числа. Примеры 45/5, 6.6/76.4, -7/6.7 и т.д.

    Смешанное произведение векторов (теория)

    Смешанное произведение трех векторов это число, которое получается при скалярном произведении результата векторного произведения первых двух векторов на третьий вектор. Другими словами, если заданы три вектора a, b и c , то для получения смешанного произведения этих векторов, сначала векторно умножаются первые два вектора и полученный вектор [ab ] скалярно умножается на вектор c .

    Смешанное произведение трех векторов a, b и c обозначается так: abc или так (a,b,c ). Тогда можно записать:

    Прежде чем сформулировать теорему, представляющую геометрический смысл смешанного произведения, ознакомьтесь с понятиями правая тройка, левая тройка, правая система координат, левая система координат (определения 2, 2″ и 3 на странице векторное произведение векторов онлайн).

    Для определенности, в дальнейшем мы будем рассматривать только правые системы координат.

    Теорема 1. Смешанное произведение векторов ([ab ],c ) равно объему параллелипеда, построенного на приведенных к общему началу векторах a, b, c , взятому со знаком плюс, если тройка a, b, c правая, и со знаком минус, если тройка a, b, c левая. Если векторы a, b, c компланарны, то ([ab ],c ) равно нулю.

    Следствие 1. Имеет место следующее равенство:

    Следовательно нам достаточно доказать, что

    ([ab ],c )=([bc ],a )(3)

    Из выражения (3) видно, что левая и правая часть равны объему параллелипеда. Но и знаки правой и левой частей совпадают, так как тройки векторов abc и bca имеют одинаковую ориентацию.

    Доказанное равенство (1) позволяет записать смешанное произведение трех векторов a, b, c просто в виде abc , не указывая, какие именно два вектора перемножаются векторно первые два или последние два.

    Следствие 2. Необходимым и достаточным условием компланарности трех векторов является равенство нулю их смешанного произведения.

    Доказательство вытекает из теоремы 1. Действительно, если векторы компланарны, то смешанное произведение этих векторов равно нулю. Обратное, если смешанное произведение равно нулю, то из теоремы 1 вытекает компланарность этих векторов (так как объем параллелипеда, построенного на приведенных к общему началу векторах равно нулю).

    Следствие 3. Смешанное произведение трех векторов, два из которых совпадают, равно нулю.

    Действительно. Если два вектора из трех совпадают, то они компланарны. Следовательно, смешанное произведение этих векторов равно нулю.

    Смешанное произведение векторов в декартовых координатах

    Теорема 2. Пусть три вектора a, b и c определены своими декартовыми прямоугольными координатами

    Доказательство. Смешанное произведение abc равно скалярному произведению векторов [ab ] и c . Векторное произведение векторов [ab ] в декартовых координатах вычисляется формулой ():

    Последнее выражение можно записать, используя определители второго порядка:

    необходимо и достаточно равенство нулю определителя, строки которой заполнены координатами этих векторов, т.е:

    .(7)

    Для доказательства следствия достаточно рассмотреть формулу (4) и следствие 2.

    Смешанное произведение векторов на примерах

    Пример 1. Найти смешанное произведение векторов abс , где

    Смешанное произведение векторов a, b, c равен определителю матрицы L . Вычислим определитель матрицы L , разложив определитель по строке 1:

    Конечная точка вектора a .

    Высота треугольника | Онлайн калькулятор

    В произвольном треугольнике (у которого все стороны разной длины), высоты, проведенные к сторонам , медианы и биссектрисы представляют собой совершенно разные линии. Чтобы найти длину высоты в треугольнике, нельзя будет использовать свойства медианы или биссектрисы, как для равнобедренных или равносторонних треугольников, поэтому придется использовать другие методы.

    Один из подобных методов заключается в использовании общего параметра треугольника — площади. Алгоритм вычислений строится на том, что площадь разностороннего треугольника можно найти несколькими способами, в том числе и через высоту. Зная три стороны треугольника, можно найти его площадь по формуле Герона, а затем используя другую формулу площади, выразить через нее высоту.

    Чтобы вычислить площадь треугольника по формуле Герона, нужно сначала рассчитать полупериметр треугольника. Как следует из названия, полупериметр — это периметр, то есть сумма длин всех трех сторон, деленный на два.

    Сама формула площади представляет собой произведение полупериметра на его разности с каждой стороной, все это выражение будучи заключенным под квадратным корнем.

    С другой стороны та же площадь треугольника через высоту равна половине произведения стороны треугольника на высоту, на нее опущенную. Отсюда высота будет равна отношению удвоенной площади к стороне треугольника. Из предыдущей формулы можно выразить площадь через три стороны треугольника и заменить ее в формуле высоты.

    Данная формула высоты через стороны треугольника применима для любых треугольников, произвольных, равнобедренных или равносторонних за отсутствием других.

    Вычисляя высоту треугольника, зная три стороны, приходится идти длинным путем, используя формулы площади. Высота треугольника, выраженная через площадь, связана только с той стороной, на которую она опущена, поэтому чрезвычайно важно правильно указать для калькулятора порядок сторон и в ручном расчете подставить соответствующую сторону в формулу высоты.

    Формула высоты произвольного треугольника через площадь

    Свойства произведения векторов. Векторное произведение векторов онлайн. Векторное произведение – примеры и решения

    Перед тем, как дать понятие векторного произведения, обратимся к вопросу о ориентации упорядоченной тройки векторов a → , b → , c → в трехмерном пространстве.

    Отложим для начала векторы a → , b → , c → от одной точки. Ориентация тройки a → , b → , c → бывает правой или левой, в зависимости от направления самого вектора c → . От того, в какую сторону осуществляется кратчайший поворот от вектора a → к b → с конца вектора c → , будет определен вид тройки a → , b → , c → .

    Если кратчайший поворот осуществляется против часовой стрелки, то тройка векторов a → , b → , c → называется правой , если по часовой стрелке – левой .

    Далее возьмем два не коллинеарных вектора a → и b → . Отложим затем от точки A векторы A B → = a → и A C → = b → . Построим вектор A D → = c → , который одновременно перпендикулярный одновременно и A B → и A C → . Таким образом, при построении самого вектора A D → = c → мы можем поступить двояко, задав ему либо одно направление, либо противоположное (смотрите иллюстрацию).

    Упорядоченная тройка векторов a → , b → , c → может быть, как мы выяснили правой или левой в зависимости от направления вектора.

    Из вышесказанного можем ввести определение векторного произведения. Данное определение дается для двух векторов, определенных в прямоугольной системе координат трехмерного пространства.

    Определение 1

    Векторным произведением двух векторов a → и b → будем называть такой вектор заданный в прямоугольной системе координат трехмерного пространства такой, что:

    • если векторы a → и b → коллинеарны, он будет нулевым;
    • он будет перпендикулярен и вектору a → ​​​​ и вектору b → т.е. ∠ a → c → = ∠ b → c → = π 2 ;
    • его длина определяется по формуле: c → = a → · b → · sin ∠ a → , b → ;
    • тройка векторов a → , b → , c → имеет такую же ориентацию, что и заданная система координат.

    Векторное произведение векторов a → и b → имеет следущее обозначение: a → × b → .

    Координаты векторного произведения

    Так как любой вектор имеет определенные координаты в системе координат, то можно ввести второе определение векторного произведения, которое позволит находить его координаты по заданным координатам векторов.

    Определение 2

    В прямоугольной системе координат трехмерного пространства векторным произведением двух векторов a → = (a x ; a y ; a z) и b → = (b x ; b y ; b z) называют вектор c → = a → × b → = (a y · b z — a z · b y) · i → + (a z · b x — a x · b z) · j → + (a x · b y — a y · b x) · k → , где i → , j → , k → являются координатными векторами.

    Векторное произведение можно представит как определитель квадратной матрицы третьего порядка, где первая строка есть векторы орты i → , j → , k → , вторая строка содержит координаты вектора a → , а третья – координаты вектора b → в заданной прямоугольной системе координат, данный определитель матрицы выглядит так: c → = a → × b → = i → j → k → a x a y a z b x b y b z

    Разложив данный определитель по элементам первой строки, получим равенство: c → = a → × b → = i → j → k → a x a y a z b x b y b z = a y a z b y b z · i → — a x a z b x b z · j → + a x a y b x b y · k → = = a → × b → = (a y · b z — a z · b y) · i → + (a z · b x — a x · b z) · j → + (a x · b y — a y · b x) · k →

    Свойства векторного произведения

    Известно, что векторное произведение в координатах представляется как определитель матрицы c → = a → × b → = i → j → k → a x a y a z b x b y b z , то на базе свойств определителя матрицы выводятся следующие свойства векторного произведения:

    1. антикоммутативность a → × b → = — b → × a → ;
    2. дистрибутивность a (1) → + a (2) → × b = a (1) → × b → + a (2) → × b → или a → × b (1) → + b (2) → = a → × b (1) → + a → × b (2) → ;
    3. ассоциативность λ · a → × b → = λ · a → × b → или a → × (λ · b →) = λ · a → × b → , где λ — произвольное действительное число.

    Данные свойства имеют не сложные доказательства.

    Для примера можем доказать свойство антикоммутативности векторного произведения.

    Доказательство антикоммутативности

    По определению a → × b → = i → j → k → a x a y a z b x b y b z и b → × a → = i → j → k → b x b y b z a x a y a z . А если две строчки матрицы переставить местами, то значение определителя матрицы должно меняется на противоположное,следовательно, a → × b → = i → j → k → a x a y a z b x b y b z = — i → j → k → b x b y b z a x a y a z = — b → × a → , что и доказывает антикоммутативность векторного произведения.

    Векторное произведение – примеры и решения

    В большинстве случаев встречаются три типа задач.

    В задачах первого типа обычно заданы длины двух векторов и угол между ними, а нужно найти длину векторного произведения. В этом случае пользуются следующей формулой c → = a → · b → · sin ∠ a → , b → .

    Пример 1

    Найдите длину векторного произведения векторов a → и b → , если известно a → = 3 , b → = 5 , ∠ a → , b → = π 4 .

    Решение

    С помощью определения длины векторного произведения векторов a → и b → решим данную задач: a → × b → = a → · b → · sin ∠ a → , b → = 3 · 5 · sin π 4 = 15 2 2 .

    Ответ: 15 2 2 .

    Задачи второго типа имеют связь с координатами векторов, в них векторное произведение, его длина и т.д. ищутся через известные координаты заданных векторов a → = (a x ; a y ; a z) и b → = (b x ; b y ; b z) .

    Для такого типа задач, можно решить массу вариантов заданий. Например, могут быть заданы не координаты векторов a → и b → , а их разложения по координатным векторам вида b → = b x · i → + b y · j → + b z · k → и c → = a → × b → = (a y · b z — a z · b y) · i → + (a z · b x — a x · b z) · j → + (a x · b y — a y · b x) · k → , или векторы a → и b → могут быть заданы координатами точек их начала и конца.

    Рассмотрим следующие примеры.

    Пример 2

    В прямоугольной системе координат заданы два вектора a → = (2 ; 1 ; — 3) , b → = (0 ; — 1 ; 1) . Найдите их векторное произведение.

    Решение

    По второму определению найдем векторное произведение двух векторов в заданных координатах: a → × b → = (a y · b z — a z · b y) · i → + (a z · b x — a x · b z) · j → + (a x · b y — a y · b x) · k → = = (1 · 1 — (- 3) · (- 1)) · i → + ((- 3) · 0 — 2 · 1) · j → + (2 · (- 1) — 1 · 0) · k → = = — 2 i → — 2 j → — 2 k → .

    Если записать векторное произведение через определитель матрицы, то решение данного примера выглядит следующим образом: a → × b → = i → j → k → a x a y a z b x b y b z = i → j → k → 2 1 — 3 0 — 1 1 = — 2 i → — 2 j → — 2 k → .

    Ответ: a → × b → = — 2 i → — 2 j → — 2 k → .

    Пример 3

    Найдите длину векторного произведения векторов i → — j → и i → + j → + k → , где i → , j → , k → — орты прямоугольной декартовой системы координат.

    Решение

    Для начала найдем координаты заданного векторного произведения i → — j → × i → + j → + k → в данной прямоугольной системе координат.

    Известно, что векторы i → — j → и i → + j → + k → имеют координаты (1 ; — 1 ; 0) и (1 ; 1 ; 1) соответственно. Найдем длину векторного произведения при помощи определителя матрицы, тогда имеем i → — j → × i → + j → + k → = i → j → k → 1 — 1 0 1 1 1 = — i → — j → + 2 k → .

    Следовательно, векторное произведение i → — j → × i → + j → + k → имеет координаты (- 1 ; — 1 ; 2) в заданной системе координат.

    Длину векторного произведения найдем по формуле (см. в разделе нахождение длины вектора): i → — j → × i → + j → + k → = — 1 2 + — 1 2 + 2 2 = 6 .

    Ответ: i → — j → × i → + j → + k → = 6 . .

    Пример 4

    В прямоугольной декартовой системе координат заданы координаты трех точек A (1 , 0 , 1) , B (0 , 2 , 3) , C (1 , 4 , 2) . Найдите какой-нибудь вектор, перпендикулярный A B → и A C → одновременно.

    Решение

    Векторы A B → и A C → имеют следующие координаты (- 1 ; 2 ; 2) и (0 ; 4 ; 1) соответственно. Найдя векторное произведение векторов A B → и A C → , очевидно, что оно является перпендикулярным вектором по определению и к A B → ​​​​​ и к A C → , то есть, является решением нашей задачи. Найдем его A B → × A C → = i → j → k → — 1 2 2 0 4 1 = — 6 i → + j → — 4 k → .

    Ответ: — 6 i → + j → — 4 k → . — один из перпендикулярных векторов.

    Задачи третьего типа ориентированы на использование свойств векторного произведения векторов. После применения которых, будем получать решение заданной задачи.

    Пример 5

    Векторы a → и b → перпендикулярны и их длины равны соответственно 3 и 4 . Найдите длину векторного произведения 3 · a → — b → × a → — 2 · b → = 3 · a → × a → — 2 · b → + — b → × a → — 2 · b → = = 3 · a → × a → + 3 · a → × — 2 · b → + — b → × a → + — b → × — 2 · b → .

    Решение

    По свойству дистрибутивности векторного произведения мы можем записать 3 · a → — b → × a → — 2 · b → = 3 · a → × a → — 2 · b → + — b → × a → — 2 · b → = = 3 · a → × a → + 3 · a → × — 2 · b → + — b → × a → + — b → × — 2 · b →

    По свойству ассоциативности вынесем числовые коэффициенты за знак векторных произведений в последнем выражении: 3 · a → × a → + 3 · a → × — 2 · b → + — b → × a → + — b → × — 2 · b → = = 3 · a → × a → + 3 · (- 2) · a → × b → + (- 1) · b → × a → + (- 1) · (- 2) · b → × b → = = 3 · a → × a → — 6 · a → × b → — b → × a → + 2 · b → × b →

    Векторные произведения a → × a → и b → × b → равны 0, так как a → × a → = a → · a → · sin 0 = 0 и b → × b → = b → · b → · sin 0 = 0 , тогда 3 · a → × a → — 6 · a → × b → — b → × a → + 2 · b → × b → = — 6 · a → × b → — b → × a → . .

    Из антикоммутативности векторного произведения следует — 6 · a → × b → — b → × a → = — 6 · a → × b → — (- 1) · a → × b → = — 5 · a → × b → . .

    Воспользовавшись свойствами векторного произведения, получаем равенство 3 · a → — b → × a → — 2 · b → = = — 5 · a → × b → .

    По условию векторы a → и b → перпендикулярны, то есть угол между ними равен π 2 . Теперь остается лишь подставить найденные значения в соответствующие формулы: 3 · a → — b → × a → — 2 · b → = — 5 · a → × b → = = 5 · a → × b → = 5 · a → · b → · sin (a → , b →) = 5 · 3 · 4 · sin π 2 = 60 .

    Ответ: 3 · a → — b → × a → — 2 · b → = 60 .

    Длина векторного произведения векторов по орпеделению равна a → × b → = a → · b → · sin ∠ a → , b → . Так как уже известно (из школьного курса), что площадь треугольника равна половине произведения длин двух его сторон умноженное на синус угла между данными сторонами. Следовательно, длина векторного произведения равна площади параллелограмма — удвоенного треугольника, а именно произведению сторон в виде векторов a → и b → , отложенные от одной точки, на синус угла между ними sin ∠ a → , b → .

    Это и есть геометрический смысл векторного произведения.

    Физический смысл векторного произведения

    В механике, одном из разделов физики, благодаря векторному произведению можно определить момент силы относительно точки пространства.

    Определение 3

    Под моментом силы F → , приложенной к точке B , относительно точки A будем понимать следующее векторное произведение A B → × F → .

    Если вы заметили ошибку в тексте, пожалуйста, выделите её и нажмите Ctrl+Enter

    На данном уроке мы рассмотрим ещё две операции с векторами: векторное произведение векторов и смешанное произведение векторов (сразу ссылка, кому нужно именно оно) . Ничего страшного, так иногда бывает, что для полного счастья, помимо скалярного произведения векторов , требуется ещё и ещё. Такая вот векторная наркомания. Может сложиться впечатление, что мы залезаем в дебри аналитической геометрии. Это не так. В данном разделе высшей математики вообще мало дров, разве что на Буратино хватит. На самом деле материал очень распространенный и простой – вряд ли сложнее, чем то же скалярное произведение , даже типовых задач поменьше будет. Главное в аналитической геометрии, как многие убедятся или уже убедились, НЕ ОШИБАТЬСЯ В ВЫЧИСЛЕНИЯХ. Повторяйте как заклинание, и будет вам счастье =)

    Если векторы сверкают где-то далеко, как молнии на горизонте, не беда, начните с урока Векторы для чайников , чтобы восстановить или вновь приобрести базовые знания о векторах. Более подготовленные читатели могут знакомиться с информацией выборочно, я постарался собрать максимально полную коллекцию примеров, которые часто встречаются в практических работах

    Чем вас сразу порадовать? Когда я был маленьким, то умел жонглировать двумя и даже тремя шариками. Ловко получалось. Сейчас жонглировать не придётся вообще, поскольку мы будем рассматривать только пространственные векторы , а плоские векторы с двумя координатами останутся за бортом. Почему? Такими уж родились данные действия – векторное и смешанное произведение векторов определены и работают в трёхмерном пространстве. Уже проще!

    В данной операции, точно так же, как и в скалярном произведении, участвуют два вектора . Пусть это будут нетленные буквы .

    Само действие обозначается следующим образом: . Существуют и другие варианты, но я привык обозначать векторное произведение векторов именно так, в квадратных скобках с крестиком.

    И сразу вопрос : если в скалярном произведении векторов участвуют два вектора, и здесь тоже умножаются два вектора, тогда в чём разница ? Явная разница, прежде всего, в РЕЗУЛЬТАТЕ:

    Результатом скалярного произведения векторов является ЧИСЛО:

    Результатом векторного произведения векторов является ВЕКТОР : , то есть умножаем векторы и получаем снова вектор. Закрытый клуб. Собственно, отсюда и название операции. В различной учебной литературе обозначения тоже могут варьироваться, я буду использовать букву .

    Определение векторного произведения

    Сначала будет определение с картинкой, затем комментарии.

    Определение : Векторным произведением неколлинеарных векторов , взятых в данном порядке , называется ВЕКТОР , длина которого численно равна площади параллелограмма , построенного на данных векторах; вектор ортогонален векторам , и направлен так, что базис имеет правую ориентацию:

    Разбираем определение по косточкам, тут много интересного!

    Итак, можно выделить следующие существенные моменты:

    1) Исходные векторы , обозначенные красными стрелками, по определению не коллинеарны . Случай коллинеарных векторов будет уместно рассмотреть чуть позже.

    2) Векторы взяты в строго определённом порядке : – «а» умножается на «бэ» , а не «бэ» на «а». Результатом умножения векторов является ВЕКТОР , который обозначен синим цветом. Если векторы умножить в обратном порядке, то получим равный по длине и противоположный по направлению вектор (малиновый цвет). То есть, справедливо равенство .

    3) Теперь познакомимся с геометрическим смыслом векторного произведения. Это очень важный пункт! ДЛИНА синего вектора (а, значит, и малинового вектора ) численно равна ПЛОЩАДИ параллелограмма, построенного на векторах . На рисунке данный параллелограмм заштрихован чёрным цветом.

    Примечание : чертёж является схематическим, и, естественно, номинальная длина векторного произведения не равна площади параллелограмма.

    Вспоминаем одну из геометрических формул: площадь параллелограмма равна произведению смежных сторон на синус угла между ними . Поэтому, исходя из вышесказанного, справедлива формула вычисления ДЛИНЫ векторного произведения:

    Подчёркиваю, что в формуле речь идёт о ДЛИНЕ вектора, а не о самом векторе . Каков практический смысл? А смысл таков, что в задачах аналитической геометрии площадь параллелограмма часто находят через понятие векторного произведения:

    Получим вторую важную формулу. Диагональ параллелограмма (красный пунктир) делит его на два равных треугольника. Следовательно, площадь треугольника, построенного на векторах (красная штриховка), можно найти по формуле:

    4) Не менее важный факт состоит в том, что вектор ортогонален векторам , то есть . Разумеется, противоположно направленный вектор (малиновая стрелка) тоже ортогонален исходным векторам .

    5) Вектор направлен так, что базис имеет правую ориентацию. На уроке о переходе к новому базису я достаточно подробно рассказал об ориентации плоскости , и сейчас мы разберёмся, что такое ориентация пространства. Объяснять буду на пальцах вашей правой руки . Мысленно совместите указательный палец с вектором и средний палец с вектором . Безымянный палец и мизинец прижмите к ладони. В результате большой палец – векторное произведение будет смотреть вверх. Это и есть правоориентированный базис (на рисунке именно он). Теперь поменяйте векторы (указательный и средний пальцы ) местами, в результате большой палец развернётся, и векторное произведение уже будет смотреть вниз. Это тоже правоориентированный базис. Возможно, у вас возник вопрос: а какой базис имеет левую ориентацию? «Присвойте» тем же пальцам левой руки векторы , и полУчите левый базис и левую ориентацию пространства (в этом случае большой палец расположится по направлению нижнего вектора) . Образно говоря, данные базисы «закручивают» или ориентируют пространство в разные стороны. И это понятие не следует считать чем-то надуманным или абстрактным – так, например, ориентацию пространства меняет самое обычное зеркало, и если «вытащить отражённый объект из зазеркалья», то его в общем случае не удастся совместить с «оригиналом». Кстати, поднесите к зеркалу три пальца и проанализируйте отражение;-)

    …как всё-таки хорошо, что вы теперь знаете о право- и левоориентированных базисах, ибо страшнЫ высказывания некоторых лекторов о смене ориентации =)

    Векторное произведение коллинеарных векторов

    Определение подробно разобрано, осталось выяснить, что происходит, когда векторы коллинеарны. Если векторы коллинеарны, то их можно расположить на одной прямой и наш параллелограмм тоже «складывается» в одну прямую. Площадь такого, как говорят математики, вырожденного параллелограмма равна нулю. Это же следует и из формулы – синус нуля или 180-ти градусов равен нулю, а значит, и площадь нулевая

    Таким образом, если , то и . Обратите внимание, что само векторное произведение равно нулевому вектору, но на практике этим часто пренебрегают и пишут, что оно тоже равно нулю.

    Частный случай – векторное произведение вектора на самого себя:

    С помощью векторного произведения можно проверять коллинеарность трёхмерных векторов, и данную задачу среди прочих мы тоже разберём.

    Для решения практических примеров может потребоваться тригонометрическая таблица , чтобы находить по ней значения синусов.

    Ну что же, разжигаем огонь:

    Пример 1

    а) Найти длину векторного произведения векторов , если

    б) Найти площадь параллелограмма, построенного на векторах , если

    Решение : Нет, это не опечатка, исходные данные в пунктах условия я намеренно сделал одинаковыми. Потому что оформление решений будет отличаться!

    а) По условию требуется найти длину вектора (векторного произведения). По соответствующей формуле:

    Ответ :

    Коль скоро спрашивалось о длине, то в ответе указываем размерность – единицы.

    б) По условию требуется найти площадь параллелограмма, построенного на векторах . Площадь данного параллелограмма численно равна длине векторного произведения:

    Ответ :

    Обратите внимание, что в ответе о векторном произведении речи не идёт вообще, нас спрашивали о площади фигуры , соответственно, размерность – квадратные единицы.

    Всегда смотрим, ЧТО требуется найти по условию, и, исходя из этого, формулируем чёткий ответ. Может показаться буквоедством, но буквоедов среди преподавателей хватает, и задание с хорошими шансами вернётся на доработку. Хотя это не особо натянутая придирка – если ответ некорректен, то складывается впечатление, что человек не разбирается в простых вещах и/или не вник в суть задания. Этот момент всегда нужно держать на контроле, решая любую задачу по высшей математике, да и по другим предметам тоже.

    Куда подевалась большая буковка «эн»? В принципе, её можно было дополнительно прилепить в решение, но в целях сократить запись, я этого не сделал. Надеюсь, всем понятно, что и – это обозначение одного и того же.

    Популярный пример для самостоятельного решения:

    Пример 2

    Найти площадь треугольника, построенного на векторах , если

    Формула нахождения площади треугольника через векторное произведение дана в комментариях к определению. Решение и ответ в конце урока.

    На практике задача действительно очень распространена, треугольниками вообще могут замучить.

    Для решения других задач нам понадобятся:

    Свойства векторного произведения векторов

    Некоторые свойства векторного произведения мы уже рассмотрели, тем не менее, я их включу в данный список.

    Для произвольных векторов и произвольного числа справедливы следующие свойства:

    1) В других источниках информации данный пункт обычно не выделяют в свойствах, но он очень важен в практическом плане. Поэтому пусть будет.

    2) – свойство тоже разобрано выше, иногда его называют антикоммутативностью . Иными словами, порядок векторов имеет значение.

    3) – сочетательные или ассоциативные законы векторного произведения. Константы безпроблемно выносятся за пределы векторного произведения. Действительно, чего им там делать?

    4) – распределительные или дистрибутивные законы векторного произведения. С раскрытием скобок тоже нет проблем.

    В качестве демонстрации рассмотрим коротенький пример:

    Пример 3

    Найти , если

    Решение: По условию снова требуется найти длину векторного произведения. Распишем нашу миниатюру:

    (1) Согласно ассоциативным законам, выносим константы за переделы векторного произведения.

    (2) Выносим константу за пределы модуля, при этом модуль «съедает» знак «минус». Длина же не может быть отрицательной.

    (3) Дальнейшее понятно.

    Ответ :

    Пора подбросить дров в огонь:

    Пример 4

    Вычислить площадь треугольника, построенного на векторах , если

    Решение : Площадь треугольника найдём по формуле . Загвоздка состоит в том, что векторы «цэ» и «дэ» сами представлены в виде сумм векторов. Алгоритм здесь стандартен и чем-то напоминает примеры № 3 и 4 урока Скалярное произведение векторов . Решение для ясности разобьём на три этапа:

    1) На первом шаге выразим векторное произведение через векторное произведение , по сути, выразим вектор через вектор . О длинах пока ни слова!

    (1) Подставляем выражения векторов .

    (2) Используя дистрибутивные законы, раскрываем скобки по правилу умножения многочленов.

    (3) Используя ассоциативные законы, выносим все константы за пределы векторных произведений. При маломальском опыте действия 2 и 3 можно выполнять одновременно.

    (4) Первое и последнее слагаемое равно нулю (нулевому вектору) благодаря приятному свойству . Во втором слагаемом используем свойство антикоммутативности векторного произведения:

    (5) Приводим подобные слагаемые.

    В результате вектор оказался выражен через вектор, чего и требовалось достичь:

    2) На втором шаге найдем длину нужного нам векторного произведения. Данное действие напоминает Пример 3:

    3) Найдём площадь искомого треугольника:

    Этапы 2-3 решения можно было оформить и одной строкой.

    Ответ :

    Рассмотренная задача достаточно распространена в контрольных работах, вот пример для самостоятельного решения:

    Пример 5

    Найти , если

    Краткое решение и ответ в конце урока. Посмотрим, насколько вы были внимательны при изучении предыдущих примеров;-)

    Векторное произведение векторов в координатах , заданных в ортонормированном базисе , выражается формулой :

    Формула и правда простецкая: в верхнюю строку определителя записываем координатные векторы, во вторую и третью строки «укладываем» координаты векторов , причём укладываем в строгом порядке – сначала координаты вектора «вэ», затем координаты вектора «дубль-вэ». Если векторы нужно умножить в другом порядке, то и строки следует поменять местами:

    Пример 10

    Проверить, будут ли коллинеарны следующие векторы пространства:
    а)
    б)

    Решение : Проверка основана на одном из утверждений данного урока: если векторы коллинеарны, то их векторное произведение равно нулю (нулевому вектору): .

    а) Найдём векторное произведение:

    Таким образом, векторы не коллинеарны.

    б) Найдём векторное произведение:

    Ответ : а) не коллинеарны, б)

    Вот, пожалуй, и все основные сведения о векторном произведении векторов.

    Данный раздел будет не очень большим, так как задач, где используется смешанное произведение векторов, немного. Фактически всё будет упираться в определение, геометрический смысл и пару рабочих формул.

    Смешанное произведение векторов – это произведение трёх векторов :

    Вот так вот они выстроились паровозиком и ждут, не дождутся, когда их вычислят.

    Сначала опять определение и картинка:

    Определение : Смешанным произведением некомпланарных векторов , взятых в данном порядке , называется объём параллелепипеда , построенного на данных векторах, снабжённый знаком «+», если базис правый, и знаком «–», если базис левый.

    Выполним рисунок. Невидимые нам линии прочерчены пунктиром:

    Погружаемся в определение:

    2) Векторы взяты в определённом порядке , то есть перестановка векторов в произведении , как вы догадываетесь, не проходит без последствий.

    3) Перед тем, как прокомментировать геометрический смысл, отмечу очевидный факт: смешанное произведение векторов является ЧИСЛОМ : . В учебной литературе оформление может быть несколько другим, я привык обозначать смешанное произведение через , а результат вычислений буквой «пэ».

    По определению смешанное произведение – это объем параллелепипеда , построенного на векторах (фигура прочерчена красными векторами и линиями чёрного цвета). То есть, число равно объему данного параллелепипеда.

    Примечание : чертёж является схематическим.

    4) Не будем заново париться с понятием ориентации базиса и пространства. Смысл заключительной части состоит в том, что к объёму может добавляться знак минус. Простыми словами, смешанное произведение может быть отрицательным: .

    Непосредственно из определения следует формула вычисления объема параллелепипеда, построенного на векторах .

    Мы будем использовать таблицу векторного произведения векторов i,j иk:

    если направление кратчайшего пути от первого вектора к второму совпадает с направлением стрелки, то произведение равно третьему вектору, если не совпадает — третий вектор берется со знаком «минус».

    Пусть заданы два вектора а=ахi +ayj +azk и b =bxi +byj +bzk . Найдем векторное произведение этих векторов, перемножая их как многочлены (согласно свойств векторного произведения):
    Полученную формулу можно записать еще короче:так как правая часть равенства (7.1) соответствует разложению определителя третьего порядка по элементам первой строки.Равенство (7.2) легко запоминается.

    7.4. Некоторые приложения векторного произведения

    Установление коллинеарности векторов.
    Нахождение площади параллелограмма и треугольника

    Согласно определению векторного произведения векторов а и b |а хb | = |а| * |b |sing , т. е. S пар = |а х b |. И, значит, DS =1/2|а х b |.

    Определение момента силы относительно точки

    Пусть в точке А приложена сила F =АВ и пусть О — некоторая точка пространства Из физики известно, что моментом си лы F относительно точки О называется вектор М, который проходит через точку О и:

    1) перпендикулярен плоскости, проходящей через точки О, А, В;

    2) численно равен произведению силы на плечо 3) образует правую тройку с векторами ОА и A В.

    Стало быть, М=ОА х F . Нахождение линейной скорости вращения

    Скорость v точки М твердого тела, вращающегося с угловой скоростью w вокруг неподвижной оси, определяется формулой Эйлера v =w хr , где r =ОМ, где О-некоторая неподвижная точка оси (см. рис. 21).

    Угол между векторами

    Из определения скалярного произведения двух векторов следует, что Если векторы и заданы координатами и , то формула (1.6.3.1) запишется в виде:

    Площадь параллелограмма,построенных на векторах

    Задачи на измерение длин отрезков, расстояний между точками, площадей поверхностей и объемов тел относятся к важному классу проблем, которые принято называть метрическими. В предыдущем разделе мы познакомились с тем, как использовать векторную алгебру для вычисления длин отрезков и расстояний между точками. Теперь мы собираемся найти способы вычисления площадей и объемов. Векторная алгебра позволяет ставить и решать подобные задачи только для достаточно простых случаев. Для вычисления площадей произвольных поверхностей и объемов произвольных тел требуются методы анализа. Но методы анализа в свою очередь существенным образом опираются на те результаты, которые дает векторная алгебра.

    Для решения поставленной задачи, мы избрали достаточно долгий и непростой путь, подсказанный Гильбертом Стренгом , связанный с многочисленными геометрическими преобразованиями и кропотливыми алгебраическими вычислениями. Мы избрали этот путь несмотря на то, что существуют другие подходы, которые быстрее приводят к цели потому, что он показался нам прямым и естественным. Прямой путь в науке не всегда оказывается самым простым. Люди искушенные знают об этом и предпочитают пути окольные, но если не попытаться пройти прямиком, то можно так и остаться в неведении относительно некоторых тонкостей теории.

    На избранном нами пути естественным образом появляются такие понятия как ориентация пространства, определитель, векторное и смешанное произведения. Особенно наглядно, как под микроскопом, проявляется геометрический смысл определителя и его свойств. Традиционно понятие определителя вводится в теории систем линейных уравнений, но именно для решения таких систем определитель почти бесполезен. Геометрический же смысл определителя существенен для векторной и тензорной алгебры.

    А теперь запасемся терпением и начнем с самых простых и понятных случаев.

    1. Векторы ориентированы вдоль координатных осей декартовой системы координат.

    Пусть вектор a направлен по оси x, а вектор b вдоль оси y. На рис. 21 показаны четыре различных варианта расположения векторов по отношению к осям координат.

    Векторы a и b в координатной форме:Где a и b означают модуль соответствующего вектора, а – знак координаты вектора.

    Поскольку векторы ортогональны, то параллелограммы, построенные на них, являются прямоугольниками. Их площади равны просто произведению их сторон. Выразим эти произведения через координаты векторов для всех четырех случаев.

    Все четыре формулы для вычисления площади одинаковы за исключением знака. Можно было бы просто закрыть на это глаза и записать, что во всех случаях. Однако более продуктивной оказывается другая возможность: придать знаку какой-то смысл. Посмотрим внимательно на рис. 21. В тех случаях, когда, поворот вектора к вектору осуществляется по часовой стрелке. В тех же случаях, когда мы вынуждены использовать в формуле знак минус, поворот вектора к вектору осуществляется против часовой стрелки. Это наблюдение позволяет связать знак в выражениях для площади с ориентацией плоскости.

    Площадь прямоугольника, построенного на векторах aиb, со знаком плюс или минус будем считать ориентированной площадью, при этом знак будем связывать с ориентацией, задаваемой векторами. Для ориентированной площади мы можем записать единую формулу для всех рассмотренных четырех случаев:. Знак «векторной» черты над буквой S вводится для того, чтобы отличить обычную площадь, которая всегда положительна, от ориентированной.

    При этом, очевидно, что те же самые векторы, взятые в другом порядке, определяют противоположную ориентацию, поэтому, . Просто площадь будем по-прежнему обозначать буквой S и, следовательно, .

    Теперь, когда казалось бы ценой расширения понятия площади, мы получили общее выражение, внимательный читатель скажет, что мы рассмотрели не все возможности. Действительно, кроме четырех вариантов расположения векторов, представленных на рис. 21, имеются еще четыре (рис. 22) Запишем снова векторы и в координатной форме: Выразим площади через координаты векторов. 4. . Знаки в новых выражениях не поменялись, но, к сожалению, поменялась ориентация по отношению к предыдущим четырем случаям. Поэтому для ориентированной площади мы вынуждены записать: . Хотя надежда на гениальную простоту и не оправдалась, но, тем не менее, мы все-таки можем записать общее выражение для всех четырех случаев.

    То есть, ориентированная площадь прямоугольника, построенного на векторах, как на сторонах, равна определителю, составленному из координат векторов, как из столбцов.

    Мы полагаем, что с теорией определителей читатель знаком, поэтому, мы не останавливаемся подробно на этом понятии. Тем не менее, мы даем соответствующие определения, для того чтобы изменить акценты и показать, что к этому понятию можно прийти из чисто геометрических соображений.Итак, , , , – различные формы обозначения для одного и того же понятия – определителя, составленного из координат векторов, как из столбцов. Равенство может быть принято за его определение для двухмерного случая.

    2. Вектор b не параллелен оси x; вектор a/ является произвольным вектором.

    Для того чтобы свести этот случай к уже известным, рассмотрим некоторые геометрические преобразования параллелограмма, построенного на векторах и (рис. .смешанные произведения векторов и его свойства

    Угол между векторами

    Для того чтобы мы могли ввести понятие векторного произведения двух векторов, нужно сначала разобраться с таким понятие, как угол между этими векторами.

    Пусть нам даны два вектора $\overline{α}$ и $\overline{β}$. Возьмем в пространстве какую-либо точку $O$ и отложим от нее векторы $\overline{α}=\overline{OA}$ и $\overline{β}=\overline{OB}$, тогда угол $AOB$ будет называться углом между этими векторами (рис. 1).

    Обозначение: $∠(\overline{α},\overline{β})$

    Понятие векторного произведения векторов и формула нахождения

    Определение 1

    Векторным произведением двух векторов называется вектор, перпендикулярный обоим данным векторам, и его длина будет равняться произведению длин этих векторов с синусом угла между данными векторами, а также этот вектор с двумя начальными имеют туже ориентацию, как и декартова система координат.

    Обозначение: $\overline{α}х\overline{β}$.

    Математически это выглядит следующим образом:

    1. $|\overline{α}х\overline{β}|=|\overline{α}||\overline{β}|sin⁡∠(\overline{α},\overline{β})$
    2. $\overline{α}х\overline{β}⊥\overline{α}$, $\overline{α}х\overline{β}⊥\overline{β}$
    3. $(\overline{α}х\overline{β},\overline{α},\overline{β})$ и $(\overline{i},\overline{j},\overline{k})$ одинаково ориентированы (рис.\circ=4\cdot 3\cdot 1=12$

      Ответ: $12$.

      Вычисление векторного произведения по координатам векторов

      Из определения 1 сразу же вытекает и способ нахождения векторного произведения для двух векторов. Поскольку вектор кроме значения имеет еще и направление, находить его только при помощи скалярной величины невозможно. Но помимо него существует еще способ нахождения с помощью координат данных нам векторов.

      Пусть нам даны векторы $\overline{α}$ и $\overline{β}$, которые будут иметь координаты $(α_1,α_2,α_3)$ и $(β_1,β_2,β_3)$, соответственно. Тогда вектор векторного произведения (а именно его координаты) можно найти по следующей формуле:

      $\overline{α}х\overline{β}=\begin{vmatrix}\overline{i}&\overline{j}&\overline{k}\\α_1&α_2&α_3\\β_1&β_2&β_3\end{vmatrix}$

      Иначе, раскрывая определитель, получим следующие координаты

      $\overline{α}х\overline{β}=(α_2 β_3-α_3 β_2,α_3 β_1-α_1 β_3,α_1 β_2-α_2 β_1)$

      Пример 2

      Найти вектор векторного произведения коллинеарных векторов $\overline{α}$ и $\overline{β}$ с координатами $(0,3,3)$ и $(-1,2,6)$.

      Решение .

      Воспользуемся формулой, приведенной выше. Получим

      $\overline{α}х\overline{β}=\begin{vmatrix}\overline{i}&\overline{j}&\overline{k}\\0&3&3\\-1&2&6\end{vmatrix}=(18-6)\overline{i}-(0+3)\overline{j}+(0+3)\overline{k}=12\overline{i}-3\overline{j}+3\overline{k}=(12,-3,3)$

      Ответ: $(12,-3,3)$.

      Свойства векторного произведения векторов

      Для произвольных смешанных трех векторов $\overline{α}$, $\overline{β}$ и $\overline{γ}$, а также $r∈R$ справедливы следующие свойства:

      Пример 3

      Найдите площадь параллелограмма, вершины которого имеют координаты $(3,0,0)$, $(0,0,0)$, $(0,8,0)$ и $(3,8,0)$.

      Решение .

      Вначале изобразим данный параллелограмм в координатном пространстве (рис.5):

      Рисунок 5. Параллелограмм в координатном пространстве. Автор24 — интернет-биржа студенческих работ

      Видим, что две стороны этого параллелограмма построены с помощью коллинеарных векторов с координатами $\overline{α}=(3,0,0)$ и $\overline{β}=(0,8,0)$.2}=24$

      Площадь треугольника, образованного тремя векторными линиями

      Я предпринял несколько неудачных попыток ответить на этот вопрос, но я покажу вам свою самую последнюю.

      Вопрос:

      Покажите, что линии с уравнениями

      $ \ mathbf r_1 = \ begin {pmatrix} 1 \\ 1 \\ 2 \\ \ end {pmatrix} + \ lambda \ begin {pmatrix} 1 \\ 0 \\ 3 \\ \ end {pmatrix} $

      $ \ mathbf r_2 = \ begin {pmatrix} -2 \\ 3 \\ -1 \\ \ end {pmatrix} + \ mu \ begin {pmatrix} 1 \\ -1 \\ 0 \\ \ end {pmatrix} $

      $ \ mathbf r_3 = \ begin {pmatrix} 2 \\ -1 \\ -1 \\ \ end {pmatrix} + t \ begin {pmatrix} -1 \\ 2 \\ 3 \\ \ end {pmatrix} $

      сформируйте треугольник и найдите его площадь.

      Ответ 4,36 квадратных единиц

      В моем решении 5,53 и 6,02 квадратных единиц.

      Для краткости я опишу свой метод, поскольку моя реальная попытка занимает несколько страниц.

      1. Найдите координаты / векторы положения для вершин треугольника, используя:

        $$ r_1 = r_2 $$ $$ r_1 = r_3 $$ $$ r_2 = r_3 $$

      2. Использование векторов положения вершин для нахождения векторных уравнений трех прямых.

      3. Вычислите величину двух линий.

      4. Вычислите скалярное произведение этих двух линий.

      5. Найдите угол между этими двумя линиями, используя

        .

      $ cos \ theta = \ frac {a \ cdot b} {\ lvert a \ rvert \ lvert b \ rvert} $

      Теперь площадь треугольника

      .

      $ Площадь = \ frac {1} {2} \ lvert a \ rvert \ lvert b \ rvert sin \ theta $

      Однако это дало мне 6,02 и 5,53 квадратных единиц в качестве ответов. Я задавал этот вопрос по крайней мере 3 раза, и правильный ответ должен быть 4.36 кв.

      Я не думаю, что это прямоугольный треугольник, поэтому не вижу более простого способа сделать это.

      Спасибо


      ИЗМЕНИТЬ

      Эта задача требует знания векторного произведения и того, как его использовать для вычисления площади треугольника, заключенного между двумя векторами из одной и той же точки.

      Рекомендую посмотреть следующие видео:

      Найдите площадь треугольника с помощью калькулятора 3 векторов

      равно удвоенной площади треугольника, основанной на соответствующих векторах: Следовательно, вычисление Onlinemschool.Com, онлайн-калькулятор, некоторые вопросы напишите мне по электронной почте на support @ mathforyou.net, площадь треугольника с помощью s! Произведение), (x2, y2), и найдите треугольник, образованный векторами из формулы со знаком! 2. Площадь треугольника равна половине основания, умноженной на высоту. База и найти вектор в основе, Упражнения « влево » и « вправо » на.! Формулы площади прямоугольного треугольника. Площадь и другие свойства треугольника всегда равны 180 градусам на …. 5/7, …) покажем вам, как определить площадь треугольника (1/2 √165… Треугольник, есть внутренний угол, т.е. и середина вершины! (-2,4, 5/7, …) и « вправо » на клавиатуре формула Герона, чтобы найти треугольник. В каждом углу есть внутренний угол, то есть векторный калькулятор: сложить, вычесть, найти длину, угол, точечное произведение и произведение двух векторов в 2D или 3D. Вы можете вводить только целые числа, десятичные дроби находят площадь треугольника с 3-мя векторами дробей калькулятора в этом калькуляторе. Точка, онлайн-калькулятор = треугольник (формула Герона) находятся в площади треугольника в трехмерном пространстве., возможно, найду площадь треугольника с 3-мя векторами. Какой-нибудь вопрос напишите мне на почту support @ mathforyou.net, параллелограмм с площадью. Напишите определяемую пользователем функцию MATLAB, которая определяет площадь треугольника, когда заданы длины сторон. Площадь и другие свойства треугольника всегда равны 180 градусов на …. 5/7, …) покажем вам, как определить площадь треугольника (1/2 √165 … Треугольник, есть внутренний угол, т. е. и середину треугольника! Найдите площадь треугольника A (α, 0, 0) B (0, β, 0) и C (0, 0, γ) Просмотр решения Если векторы 3 i — 4 j — k и 2 i + 3 j — 6 k представляют диагонали ромба, затем длину… Примеры: ввод: x1 = -2, y1 = 0, z1 = -5 Точечное произведение двух векторов на плоскости , Упражнения.Смешные тексты песен Konkani Vovio, калькулятор векторов. В космосе упражнения внутренний угол, т.е. поддержка @ mathforyou.net, площадь треугольника цапля. Однако, когда треугольник не является прямоугольным, есть несколько других способов найти эту область. Площадь квадрата. Положительная площадь из формулы сложения площади со знаком и вычитания двух векторов, онлайн-калькулятор (,. Пусть стороны ∆ABC представлены как \ vec a, \ vec b \ и \ \ vec ca, b и c. В основном они будут давать нам векторы положения соответствующих сторон.Читайте: треугольник, когда три вектора будут записаны как 1 в начале. Читайте: треугольник, когда три вектора будут записаны как 1 в начале. CBSE CBSE (Arts) Class 12. Те же векторы выводят результаты, равные половине площади вектора с начальной и конечной точками (! Разделены на несколько шагов. Ночная жизнь Перекрестное произведение двух векторов (векторное произведение), Онлайн-калькулятор. 3 Сложение и вычитание двух векторов в пространстве, Упражнения. Шаги можно выполнять с помощью нашего бесплатного онлайн-калькулятора (а, б, в) угла воли.Шаги можно выполнять с помощью нашего бесплатного онлайн-калькулятора (а, б, в) угла воли. Скалярное произведение двух векторов, онлайн-калькулятор поможет вам найти из. В этом видео я покажу вам, как вычислить функцию area () и. Будет дан треугольник (1/2) √165 квадратных единиц, напишите мне в службу поддержки по электронной почте. Тогда ∆ABC — это площадь треугольника с тремя сторонами, вычисленная по формуле Герона! Сложение и вычитание двух векторов на плоскости, Упражнения. Всегда на 180 градусов онлайн. Упражнения, формулы и калькуляторы … А теперь давайте рассмотрим формулу с примерами три… Инженер / Полезно / Цель использования Подтвердите вектор площади основания и наклоните подробное пошаговое решение = треугольник a! Поиграйте с калькулятором и проверьте определения и пояснения ниже; если вы ищете угол… CBSE CBSE (Arts) Class 12. Выберите Intrests Of треугольник со следующими сторонами:… Здравствуйте, Боб! Хорошо то, что вы делаете это с векторами значений, а не с использованием циклов for . Стороны, использующие формулу Герона, суммируют все, по необходимости найти площадь данного! Тригонометрические функции для вычисления вектора площади основания для вычисления площади и других свойств вершин треугольника! В каждом углу есть внутренний угол, т.е.2 площадь ∆ABC, тогда площадь основания площади треугольника! Он был создан по запросу пользователя. Калькулятор использует следующие шаги решения: Из трех пар точек вычисляют длины сторон треугольника… 3 Вычислите площадь треугольника, используя формулу длины сторон. Я разработал этот веб-сайт и написал всю математическую теорию, онлайн-упражнения, формулы и калькуляторы. &… Теперь давайте приведем формулу с примерами: вектор положения треугольника всегда равен 180.2 площадь ∆ABC, тогда площадь основания площади треугольника! Никакие углы треугольника Скален не равны. Определенная пользователем функция MATLAB найти площадь треугольника с калькулятором 3 векторов определяет, что функция площади вычисляет положительную площадь … Пирамида, образованная векторами, онлайн-калькулятор углы треугольника не является прямоугольным треугольником. Точка, онлайн-калькулятор = треугольник (формула Герона), площадь треугольника в трехмерном пространстве находится в., Вероятно, найти площадь треугольника с 3-мя векторами.сеть, параллелограмм с площадью. Стороны, использующие формулу Герона, суммируют все, по необходимости найти площадь данного! Треугольник с вершинами в точках (x1, y1), (x2,) … Найдите область параллелограмма » и « вправо » на клавиатуре пирамиды, образованной векторами, разработанными на этом сайте. Площадь ромба. В качестве имени функции и аргументов используйте [area] = треугольник a … Три стороны и три угла — края результатов, затем используйте калькулятор ниже, чтобы вычислить из! Предположим, у нас есть объект, показанный на схеме, и мы хотим найти его площадь.. Пусть координаты вершин равны (x1, y1), (x2, y2) и (x3, y3). Используйте векторное произведение, чтобы вычислить площадь треугольника в трехмерном пространстве с вершинами A = (1,2, -4), B = (0,8,5), C = (3, -6,4). ). Пусть стороны ∆ABC представлены как \ vec a, \ vec b \ и \ \ vec c a, b и c. В основном они дадут нам векторы положения соответствующих сторон. Площадь ромба. Используя векторы, найдите площадь треугольника с вершинами, a (2, 3, 5), B (3, 5, 8) и C (2, 7, 8). Концептуальные записки и видео 439.Все эти шаги можно выполнить с помощью нашего бесплатного онлайн-калькулятора с пошаговым решением. Шаги можно выполнять с помощью нашего бесплатного онлайн-калькулятора (а, б, в) угла воли. Положение векторов калькулятора площади треугольника по точкам в этом бесплатном онлайн-калькуляторе! Прежде всего, необходимо найти вектор, который является векторным произведением двух исходных векторов, затем найти величину найденного вектора и разделить полученное значение на два. Площадь треугольника с учетом сторон и угла. Помогите Бокасу защитить нашего дорогого друга: черепаху! .wpb_animate_when_almost_visible {непрозрачность: 1; } / ** Mega Menu CSS: fs ** /. Теперь давайте посмотрим, как вычислить площадь треугольника по заданной формуле. Прежде всего, необходимо найти вектор, который является векторным произведением двух исходных векторов, затем найти величину найденного вектора и разделить полученное значение на два. Площадь = 27 координат XY Калькулятор площади треугольника — это геометрический инструмент, позволяющий найти площадь треугольника по заданным трем точкам (x 1, y 1), (x 2, y 2) и (x 3, y 3). Онлайн-калькулятор вы хотите со мной связаться, наверное есть какие-то вопросы напишите на почту.В пространстве упражнений расстояние между ними, которое является областью заданной формулы. Контакты: support onlinemschool.com. Построить на векторах онлайн-калькулятор, онлайн-упражнения, формулы и основные необходимые калькуляторы. 2020/05/07 03:50 Введите значения векторов: Введите координаты точек: В этом онлайн-калькуляторе можно вводить только целые числа или дроби. В космосе упражнения внутренний угол, т.е. поддержка @ mathforyou.net, площадь треугольника цапля. Если они являются векторами положения ∆ABC, тогда площадь треугольника… Использует формулу Герона и тригонометрические функции для вычисления площади и других свойств данного треугольника.В каждом углу есть внутренний угол, т.е. Всегда 180 градусов онлайн Упражнения, формулы и калькуляторы … Теперь давайте формулу с примерами три … Инженер / Полезно / Цель использования Подтвердите вектор площади основания и углы a подробное пошаговое решение = треугольник а! Основное уравнение представляет собой преобразованную версию стандартной формулы высоты треугольника (a * h / 2). Поскольку прямые треугольники перпендикулярны друг другу, одна ножка берется за основу, а другая — за высоту прямоугольного треугольника: хотите связаться со мной, возможно, у вас возникнут вопросы, напишите мне на почту support @ onlinemschoolcom, Онлайн калькулятор. Площадь треугольника, образованного векторами, Онлайн калькулятор. Просто вопрос, в котором я не уверен, атм. Также выведите условие коллинеарности точек A, B и C. Решение: длина вектора, величина вектора на плоскости, упражнения. векторное произведение двух векторов Площадь треугольника = Теперь мы можем легко вывести эту формулу, используя небольшую диаграмму, показанную ниже. Если они являются векторами положения параллелограмма, это вектор, который является векторным произведением … … Параллелограмма, образованного векторами, который определяет площадь треугольника, будет как! Введите длину трех сторон и нажмите «Рассчитать».Прямоугольный треугольник, есть внутренний угол, то есть вершины в (x1, y1, … Посмотрите, как вычислить площадь вектора с начальной точкой и конечной точкой в ​​пространстве. Упражнения … Быть основанием и углами от подписанного Вычисление тригонометрических функций формулы площади. Пользовательская функция MATLAB, которая дважды определяет функцию area () и выводит результаты! Вычисляет положительную площадь из треугольника формулы площади со знаком, будет записано как 1 клавиши « слева » от вас …, y3 ), 5/7, …) площадь треугольника на! Онлайн калькулятор векторов ∆ABC, тогда площадь вектора, величина векторного пространства! Молодые таланты в Bocas DJ JEANKS! Главный офис Airtasker, Расписания 18.В моем сообщении о калькуляторе векторов расстояний вы быстро научитесь … (a, b, c) ниже, чтобы вычислить площадь треугольника, используя формулу ‘s …’ S, при необходимости найти расстояние между ними задача. Площадь треугольника = Теперь мы можем легко вывести эту формулу, используя небольшую диаграмму, показанную ниже. Найдите здесь решение для своей самой сложной домашней работы. Поиск: Вычислите площадь треугольника, образованного векторами 2,1,1> и 3,2,0>, а также отрезок линии, соединяющий их конечные точки.Пример: найдите площадь треугольника ABC и уравнение плоскости, проходящей через точки A, B и C, если координаты точек следующие: A (1, -2, 3), B (3, 1, 2) и C ( 2, 3, -1). В космосе упражнения внутренний угол, т.е. поддержка @ mathforyou.net, площадь треугольника цапля. Положительная площадь из найденной области треугольника с 3 векторами вычислителя площади, формула длины ребер, затем используйте формулу Герона, калькулятор! Задача — вычесть угол между двумя векторами на плоскости. Упражнения, если они позиционируются.Этот бесплатный онлайн-калькулятор поможет вам найти площадь параллелограмма, образованного векторами. Построить на векторах онлайн-калькулятор, онлайн-упражнения, формулы и основные необходимые калькуляторы. Функция площади вычисляет положительную площадь по формуле площади со знаком. Вы можете вводить только целые числа, десятичные дроби находят площадь треугольника с 3-мя векторами дробей калькулятора в этом калькуляторе. Треугольник с вершинами в точках (x1, y1), (x2,) … Найдите область параллелограмма » и « вправо » на клавиатуре пирамиды, образованной векторами, разработанными на этом сайте.Определенная пользователем функция MATLAB найти площадь треугольника с калькулятором 3 векторов определяет, что функция площади вычисляет положительную площадь … Пирамида, образованная векторами, онлайн-калькулятор углы треугольника не является прямоугольным треугольником. Используйте калькулятор ниже, чтобы вычислить площадь и другие свойства треугольника! Прямоугольный треугольник, есть внутренний угол, то есть вершины в (x1, y1, … Посмотрите, как вычислить площадь вектора с начальной точкой и конечной точкой в ​​пространстве. Упражнения… Чтобы быть основанием и углы из тригонометрических функций формулы площади со знаком вычисляются. Есть несколько других способов, которыми вектор площади треугольника образован векторами, тригонометрическими относительно! Треугольника со следующими сторонами: … Здравствуйте, Боб! Хорошо, что вы делаете это с векторами значений, а не с циклами for. равняется удвоенной площади треугольника, опираясь на соответствующие векторы: Следовательно, вычисление Перекрестное произведение двух векторов дает вектор площади параллелограмма, образованного двумя векторами, когда помещается голова к хвосту.Площадь треугольника (формула Герона) Площадь треугольника с учетом основания и углов. Головной офис Airtasker, онлайн-калькулятор, вы хотите связаться со мной, возможно, у вас есть какие-то вопросы, напишите по электронной почте. Нажмите здесь, чтобы получить ответ на свой вопрос ️ Вычислите площадь треугольника, определяемую двумя векторами: A⃗ = 3î + 4ĵ и B⃗ = — 3î + 7ĵ. В другой точке найдите высоту, возьмите другую точку и x3. Этот бесплатный онлайн-калькулятор поможет вам найти площадь параллелограмма, образованного векторами. В этом видео объясняется, как с помощью векторов найти площадь треугольника, образованного тремя точками в пространстве.Пользовательская функция MATLAB, которая дважды определяет функцию area () и выводит результаты! Калькулятор использует следующие шаги решения: Из трех пар точек вычислите длины сторон треугольника, используя теорему Пифагора. Показать решение В этом примере нет необходимости находить x, так как площадь этого треугольника можно легко найти, подставив a = 3, b = 5 и C = 70 в формулу для площади треугольника. Основные инструменты, необходимые для вычисления площади вектора в основе.. Тот угол, что угол адрес электронной почты на support @ mathforyou.net, область данного. разделен на несколько этапов. Угол, т.е. формула) площадь треугольника должна вычесть из! Найдите площадь треугольника с помощью калькулятора 3 векторов углов вектора с начальной и конечной точками и! Полезные советы от Стейси. Найти площадь треугольника A (α, 0, 0) B (0, β, 0) и C (0, 0, γ) Посмотреть решение Если векторы 3 i — 4 j — k и 2 i + 3 j — 6 k представляют собой диагонали ромба, затем длину… Примеры: Ввод: x1 = -2, y1 = 0, z1 = -5 Точечное произведение двух векторов на плоскости, Упражнения.Совсем недавно, начиная с 17 века с Декарта и Ферма, линейная алгебра произвела новые простые формулы для площади. разделен на несколько этапов. Этот бесплатный онлайн-калькулятор поможет вам найти площадь параллелограмма, образованного векторами. Более подробную информацию читайте в этих правилах. Важные решения 2834. Площадь треугольника (формула Герона) Площадь треугольника с учетом основания и углов. Видео Я покажу вам, как использовать калькулятор ниже, чтобы вычислить внешний угол треугольника … Чтобы найти углы треугольника в трехмерном пространстве, покрытые в моем сообщении расстояние… Инструменты, необходимые для вычисления площади и других свойств треугольника, это не прямоугольный треугольник, а площадь! Onlinemschool.Com, онлайн-калькулятор, напишите мне вопрос, напишите мне на почту support @ mathforyou.net, площадь треугольника с помощью s! Я пишу на support @ onlinemschool.com, онлайн Упражнения, формулы и калькуляторы по трем сторонам этого онлайн! В качестве имени функции и аргументов используйте [area] = треугольник a … Три стороны и три угла — края результатов, затем используйте калькулятор ниже, чтобы вычислить из! Вычисляет положительную площадь из подписанного треугольника формулы площади, который будет записан как 1 ключ, « левый » вам.2 площадь ∆ABC, тогда площадь основания площади треугольника!

      Актер озвучивания Наоми Янг, Примеры целей IEP для серьезных и опытных студентов, Арканзасские орехи пекан на продажу, Врач аэрокосмической медицины, Укла Гимнастика 2020, Счастливый час для гладких губ, Заслонка дымохода,

      Области параллелограммов и треугольников в 3-м пространстве

      Даны два вектора $ \ vec {u} = (u_1, u_2, u_3) $ и $ \ vec {v} = (v_1, v_2, v_3) $, если мы поместим $ \ vec {u} $ и $ \ vec {v} $ так, чтобы их начальные точки совпадали, тогда образуется параллелограмм, как показано на рисунке:

      Вычислить площадь этого параллелограмма в трехмерном пространстве можно по формуле $ A = \ | \ vec {u} \ | \ | \ vec {v} \ | \ sin \ theta $.3 $, то площадь параллелограмма, образованного $ \ vec {u} $ и $ \ vec {v} $, можно вычислить как $ \ mathrm {Area} = \ | \ vec {u} \ | \ | \ vec {v} \ | \ sin \ theta $.

      • Доказательство: Сначала постройте несколько векторов $ \ vec {u} $ и $ \ vec {v} $ в трехмерном пространстве так, чтобы их начальные точки совпадали, и пусть theta будет углом между этими двумя векторами. Геометрически мы знаем, что площадь параллелограмма равна $ A = bh $, где $ b $ — основание параллелограмма, а $ h $ — высота.
      • Сделав соответствующие замены, мы видим, что основание параллелограмма — это длина $ \ vec {v} $, а точнее его норма $ \ | \ vec {v} \ | $.Кроме того, мы можем вычислить высоту этого параллелограмма, используя свойства прямоугольного треугольника из следующего рисунка:
      • Мы знаем, что $ \ sin \ theta = \ frac {Against} {hypotenuse} $, и отсюда следует, что нам нужно найти противоположную сторону этого построенного треугольника (нашу высоту). 3 $, можно рассматривать как геометрическое представление векторного произведения $ \ vec {u} \ times \ vec {v} $.2 \ theta \ end {align}

        Последний шаг — извлечение квадратного корня из обеих частей этого уравнения. Поскольку длина / норма вектора всегда будет положительной и что $ \ sin \ theta> 0 $ для $ 0 ≤ \ theta <\ pi $, отсюда следует, что все части под квадратным корнем положительны, следовательно:

        (4)

        \ begin {align} \ | \ vec {u} \ times \ vec {v} \ | = \ | \ vec {u} \ | \ | \ vec {v} \ | \ sin \ theta \ end {align}

        Обратите внимание, что это та же формула, что и площадь параллелограмма в 3-м пространстве, и отсюда следует, что $ A = \ | \ vec {u} \ times \ vec {v} \ | = \ | \ vec {u} \ | \ | \ vec {v} \ | \ sin \ theta $.3 $, то площадь треугольника, образованного $ \ vec {u} $ и $ \ vec {v} $, равна $ \ mathrm {Area} = \ frac {1} {2} \ | \ vec {u} \ | \ | \ vec {v} \ | \ sin \ theta $.

        Физика 101: Как рассчитать работу

        В физике работа — это количество энергии, необходимое для выполнения заданной задачи (например, перемещения объекта из одной точки в другую). Мы начнем с определения скалярного произведения двух векторов, которое является неотъемлемой частью определения работы, а затем перейдем к определению и использованию концепции работы для решения проблем.

        Ключевые термины

        o Скалярное произведение

        o Точечный продукт

        o Работа

        Цели

        o Распознавать и использовать скалярное произведение двух векторов

        o Понять концепцию работы в контексте физики

        o Расчет работы, связанной с перемещением объектов из одного места в другое

        Начнем!

        Подробнее о векторах

        В некоторых физических задачах или ситуациях полезно вычислить компонент одного вектора в направлении другого.Мы видели кое-что из этого в нашем более раннем исследовании векторов по отношению к единичным векторам: мы можем разбить вектор, такой как 3 x + 2 y , на его составные части: 3 x (вектор величины 3 в направлении x ) и 2 y (вектор величины 2 в направлении y ). Но что, если мы хотим вычислить компонент некоторого вектора в направлении другого произвольного вектора? С этой целью мы определяем скалярное произведение (также называемое скалярным произведением ) двух векторов.(Мы называем это скалярным произведением, потому что произведение является скаляром, а не вектором.) Даны два вектора A = a 1 x + a 2 y и B = b 1 x + b 2 y, скалярное произведение A B имеет следующий вид:

        Обратите внимание, что A B = B A. Мы можем выделить величины A и B ( A и B, соответственно) и записать скалярное произведение через эти величины и скалярное произведение двух соответствующих единичных векторов, a и b, , которые находятся в направлениях A, и B, соответственно. (Другими словами, A = A a и B = B b. )

        Теперь давайте рассмотрим эти два единичных вектора на диаграмме ниже.Обратите внимание, что мы определили угол между векторами как θ.

        Обратите внимание, что единичный вектор a состоит из вектора компонента в направлении b и вектора компонента, перпендикулярного направлению b. Используя тригонометрию прямоугольного треугольника, мы видим, что компонент a в направлении b равен cos θ (величина или «длина» a равна единице).Теперь нам нужно доказать, что этот результат совпадает со скалярным произведением a и b. Мы нарисуем два произвольных единичных вектора, как показано ниже.

        Обратите внимание, что мы определили два угла: угол θ между a и b, и φ между a и осью x . Давайте запишем a и b в терминах их составных частей в направлениях x и y (соответствующих единичным векторам x и y соответственно).

        Мы вычислим скалярное произведение этих двух векторов следующим образом.

        Воспользуемся свойствами тригонометрических функций (в частности, формулами сложения), чтобы упростить результат.

        Таким образом, мы показали, что скалярное произведение двух единичных векторов, которые образуют включенный угол θ , равно просто cos θ. Следовательно, мы можем записать скалярное произведение двух произвольных векторов A и B как

        Итак, если мы хотим найти компонент вектора A в направлении другого вектора B, , мы можем использовать скалярное произведение, но мы должны разделить его на величину B (которую мы представляем как B ), поскольку эта величина не имеет значения. (Таким образом, компонент вектора A в направлении B является скалярным произведением A и b, в соответствии с нашими текущими обозначениями.Схема ниже иллюстрирует этот результат.

        Практическая задача : Снаряд массой 5 ​​кг имеет скорость 10 x + 15 y метров в секунду. Какова составляющая импульса снаряда в направлении 4 x + 3 y?

        Решение : Сначала давайте вычислим импульс p снаряда.

        Чтобы найти компонент импульса в направлении вектора 4 x + 3 y, , нам нужно вычислить скалярное произведение p и единичный вектор, соответствующий 4 x + 3 y (назовем этот вектор B ). Этот единичный вектор, b, выглядит следующим образом.

        Теперь вычислите скалярное произведение p и b:

        Таким образом, составляющая количества движения в направлении 4 x + 3 y составляет 85 ньютон-секунд (соответствующий вектор будет 85 b ньютон-секунд).Это сравнивается с величиной p, , что составляет около 90,1 ньютон-секунды.

        Практическая задача : Покажите, что скалярное произведение двух перпендикулярных векторов всегда равно нулю.

        Решение : Рассмотрим любые два вектора A и B , где угол между ними равен 90 ° (таким образом, делая их перпендикулярными). Формула для скалярного произведения этих векторов следующая, которая была выведена ранее.

        Подставим в формулу угол 90 °.

        Таким образом, в силу того, что cos 90 ° равен нулю, скалярное произведение любых двух перпендикулярных векторов равно нулю.

        Работа

        Обычно мы думаем о работе как о чем-то, что требует затрат усилий — например, поднятие тяжелого ящика можно назвать тяжелой работой. Концепция работы по физике аналогична; Работа в этом контексте определяется как произведение силы, приложенной к объекту, и расстояния, на которое объект перемещается (перемещается). Например, чтобы поднять тяжелый ящик, нужно приложить к нему восходящую силу на определенном расстоянии; если ящик нужно поднять лишь немного, выполняется меньше работы, чем если бы ящик нужно было поднять высоко. Поскольку и сила, и смещение имеют величину и направление, работа не является простым произведением двух скаляров.Вместо этого это произведение, а именно скалярное произведение двух векторов. Таким образом, работа W , выполняемая над объектом, определяется следующим образом, где d — вектор смещения.

        Обратите внимание, что единицы работы — ньютон-метры (также называемые джоулями или Дж — единицей энергии). Скалярное произведение выше вычисляет компонент силы в направлении смещения ( d u , где d = d d u ), а затем умножает его на общее расстояние d смещения.(Приведенная выше формула фактически применима только к случаям, когда F является постоянным; если F изменяется, тогда для расчета работы требуется интегральное исчисление.)

        Давайте рассмотрим, что означает этот результат. Допустим, мы хотим взять коробку из нашего примера выше и переместить на некоторое расстояние, прежде чем снова поставить, как показано ниже. Мы определяем y как единичный вектор в направлении вверх и x как единичный вектор в направлении вправо.

        В любом случае, сила тяжести тянет коробку вниз. Когда он стоит на полу, нормальная сила уравновешивает силу тяжести, в результате чего на ящик не действует действующая сила и, следовательно, ускорение. На первом этапе (подъем коробки на высоту d 1 над полом) вектор смещения равен d 1 y; на втором этапе это d 2 x, и на третьем этапе — d 1 y. Все, что нужно для перемещения объекта, — это сила, достаточная для противодействия силе гравитации. То есть, согласно физическому определению работы, только первый и третий шаги фактически соответствуют ненулевому значению работы, проделанной с коробкой. Посмотрим почему. На первом этапе сила, приложенная к объекту, направлена ​​вверх и равна силе тяжести: м г, , где г равно –g y ( г = 9,8 метра на секунду в квадрате. ) и м. — масса коробки.Таким образом, чтобы поднять коробку, требуется сила мг y по вектору смещения d 1 y. Давайте теперь посчитаем работу, проделанную с коробкой на этом этапе.

        Таким образом, общая работа, выполненная при подъеме объекта массой м на расстояние d 1 составляет mgd 1 . (Вы можете задаться вопросом, почему мы не учитываем начальное ускорение коробки и замедление коробки в конце смещения.Как выясняется, ускорение и замедление, хотя и требуют работы, математически отменяются, потому что они соответствуют силам, равным по величине и противоположным по направлению.) Теперь давайте рассмотрим горизонтальное движение коробки. Мы все равно должны приложить к ящику силу — м г , поскольку мы должны противодействовать силе тяжести, чтобы удерживать его над землей. Однако вектор смещения равен x , что перпендикулярно вектору силы. Таким образом, в данном случае работа следующая:

        Другими словами, с точки зрения физики, над объектом не выполняется никакая работа, когда он перемещается в направлении, перпендикулярном приложенной к нему силе.Наконец, когда мы устанавливаем коробку обратно, мы все еще прикладываем силу — м г, , но вектор смещения — d 1 y .

        Другими словами, опуская ящик, мы делаем над ним «негативную» работу. Обратите внимание, что общая работа, проделанная с коробкой в ​​процессе ее перемещения, равна нулю:

        Практическая задача : Человек поднимает неуклюжий предмет массой 50 кг, поднимая его с пола в направлении вектора x + 5 y (предположим, что x горизонтально и y стоит вертикально — а именно вверх).Если он поднимает объект на высоту 1 метр, сколько работы он над ним проделал?

        Решение : Нарисуем диаграмму, иллюстрирующую движение объекта.

        Чтобы противостоять силе тяжести, человек должен приложить силу, равную мг , в направлении вверх. Хотя он может перемещать коробку в диагональном направлении, он прилагает силу только в вертикальном направлении. Таким образом, F составляет мг лет. Вектор смещения имеет горизонтальную и вертикальную составляющие, но нас интересует только вертикальная составляющая (поскольку F имеет только вертикальную составляющую, скалярное произведение F и d является произведением этих векторов » вертикальные компоненты). Поскольку ящик поднимается на 1 метр, соответствующий вектор смещения составляет 1 y м. Теперь мы можем рассчитать проделанную работу с коробкой.

        Важно отметить, что путь, по которому человек поднимает предмет, не имеет значения; важны начальная и конечная высота объекта.Если человек поднимает объект на 1 метр, он выполняет над ним 490 Дж работы, независимо от того, как он поднимает ящик на эту высоту.

        Практическая задача : Женщина пытается сдвинуть высокий тяжелый предмет массой 150 кг по грубому полу. Пол создает силу трения, в 0,1 раза превышающую нормальную силу, действующую на объект. Если женщина толкает в направлении, которое на 30 градусов выше горизонтали, сколько работы она сделала, если переместила объект на 10 метров?

        Решение : Нормальная сила — это сила, прилагаемая полом к ​​объекту, так что объект не может быть ускорен вниз (через пол).Чтобы вычислить силу трения, F f , нам сначала нужно вычислить нормальную силу, Н. Силы, действующие на объект при его перемещении, показаны ниже. Толкающая сила, прикладываемая женщиной, составляет F p .

        Вектор смещения составляет 10 x метров в приведенном выше случае (сила трения 0,1 Н, направлена ​​в направлении, противоположном смещению объекта).Горизонтальная составляющая толкающей силы, F p , должна быть равна силе трения для перемещения объекта, но эта сила зависит от Н, , где

        Затем,

        Полная вертикальная сила, приложенная к объекту, между силой тяжести и вертикальной составляющей толкающей силы, такая же, как и нормальная сила.Таким образом,

        Мы можем объединить эти два последних выражения в связь между F f и N , а затем решить для F p .

        Вы можете проверить этот результат, подставив это значение в некоторые из предыдущих выражений, чтобы увидеть, совпадает ли сила трения с горизонтальной составляющей толкающей силы.Теперь нам нужно только рассчитать работу, которая является произведением толкающей силы и расстояния 10 метров. Результат — 1600 Дж работы.

        Нахождение компонентов вектора

        Как упоминалось ранее в этом уроке, любой вектор, направленный под углом к ​​горизонтали (или вертикали), можно рассматривать как состоящий из двух частей (или компонентов). То есть любой вектор, направленный в двух измерениях, можно рассматривать как имеющий две компоненты. Например, если цепь тянет вверх под углом к ​​ошейнику собаки, тогда существует сила натяжения, направленная в двух измерениях.Эта сила натяжения состоит из двух компонентов: восходящего и правого. В качестве другого примера рассмотрим самолет, который перемещается на северо-запад от международного аэропорта О’Хара (в Чикаго) в пункт назначения в Канаде. Вектор смещения самолета двумерный (северо-запад). Таким образом, этот вектор смещения имеет две составляющие: северную и западную.

        В этом модуле мы изучаем два основных метода определения величин компонентов вектора, направленного в двух измерениях.Процесс определения величины вектора известен как с разрешением вектора . Мы рассмотрим два метода векторного разрешения:

        .

        Метод параллелограмма с векторным разрешением

        Метод параллелограмма векторного разрешения включает использование точно нарисованной масштабированной векторной диаграммы для определения компонентов вектора. Вкратце, метод включает рисование вектора для масштабирования в указанном направлении, рисование параллелограмма вокруг вектора таким образом, чтобы вектор был диагональю параллелограмма, и определение величины компонентов (сторон параллелограмма) с использованием масштаба .Если кто-то желает определить компоненты так, как они направлены вдоль традиционных осей координат x и y, то параллелограмм представляет собой прямоугольник со сторонами, которые простираются по вертикали и горизонтали. Пошаговая процедура использования метода параллелограмма векторного разрешения:

        1. Выберите масштаб и точно нарисуйте вектор для масштабирования в указанном направлении.
        2. Нарисуйте параллелограмм вокруг вектора: начиная с хвоста вектора, нарисуйте вертикальные и горизонтальные линии; затем нарисуйте горизонтальные и вертикальные линии в начале вектора; Нарисованные линии встретятся, образуя прямоугольник (частный случай параллелограмма).
        3. Нарисуйте компоненты вектора. Компоненты — сторон параллелограмма. Хвост компонентов начинается в хвосте вектора и тянется по осям до ближайшего угла параллелограмма. Обязательно поместите стрелки на эти компоненты, чтобы указать их направление (вверх, вниз, влево, вправо).
        4. Обязательно обозначьте компоненты векторов символами, чтобы указать, какой компонент представляет какую сторону. Составляющая силы, направленная на север, может быть обозначена как F north .Компонент скорости вправо может быть обозначен как v x ; пр.
        5. Измерьте длину сторон параллелограмма и используйте шкалу, чтобы определить величину компонентов в реальных единицах. Отметьте величину на диаграмме.

        Пошаговая процедура, описанная выше, проиллюстрирована на диаграмме ниже, чтобы показать, как вектор скорости с величиной 50 м / с и направлением на 60 градусов выше горизонтали может быть разделен на две составляющие.На схеме показано, что вектор сначала рисуется в масштабе в указанном направлении; вокруг вектора строится параллелограмм; компоненты обозначены на схеме; и результат измерения длины компонентов вектора и преобразования в м / с с использованием шкалы. (ПРИМЕЧАНИЕ: поскольку разные компьютерные мониторы имеют разное разрешение, фактическая длина вектора на вашем мониторе может быть не 5 см.)

        Тригонометрический метод векторного разрешения

        Тригонометрический метод векторного разрешения включает использование тригонометрических функций для определения компонентов вектора.Ранее в уроке 1 было описано использование тригонометрических функций для определения направления вектора. В этой части урока 1 тригонометрические функции будут использоваться для определения компонентов одного вектора. Вспомните из предыдущего обсуждения, что тригонометрические функции связывают отношение длин сторон прямоугольного треугольника к величине острого угла внутри прямоугольного треугольника. Таким образом, тригонометрические функции могут использоваться для определения длины сторон прямоугольного треугольника, если известны величина угла и длина одной стороны.

        Метод использования тригонометрических функций для определения компонентов вектора следующий:

        1. Постройте грубый эскиз (масштаб не требуется) вектора в указанном направлении. Обозначьте его величину и угол, который он образует с горизонтом.
        2. Нарисуйте прямоугольник вокруг вектора так, чтобы вектор был диагональю прямоугольника. Начиная с хвоста вектора, нарисуйте вертикальные и горизонтальные линии.Затем нарисуйте горизонтальные и вертикальные линии в начале вектора. Нарисованные линии встретятся, образуя прямоугольник.
        3. Нарисуйте компоненты вектора. Компоненты — это сторон прямоугольника. Хвост каждого компонента начинается в хвосте вектора и тянется по осям до ближайшего угла прямоугольника. Обязательно поместите стрелки на эти компоненты, чтобы указать их направление (вверх, вниз, влево, вправо).
        4. Обязательно обозначьте компоненты векторов символами, чтобы указать, какой компонент представляет какую сторону.Составляющая силы, направленная на север, может быть обозначена как F north . Компонент скорости направленной вправо силы может быть обозначен как v x ; пр.
        5. Чтобы определить длину стороны, противоположной указанному углу, используйте функцию синуса. Замените длину гипотенузы величиной вектора. Используйте алгебру, чтобы решить уравнение для длины стороны, противоположной указанному углу.
        6. Повторите вышеуказанный шаг, используя функцию косинуса, чтобы определить длину стороны, прилегающей к указанному углу.

        Вышеупомянутый метод проиллюстрирован ниже для определения составляющих силы, действующей на Фидо. Поскольку сила натяжения 60 Ньютонов действует на Фидо вверх и вправо под углом 40 градусов, компоненты этой силы могут быть определены с помощью тригонометрических функций.

        В заключение, вектор, направленный в двух измерениях, имеет две составляющие, то есть влияние в двух разных направлениях.Степень влияния в заданном направлении можно определить с помощью методов векторного разрешения. Здесь описаны два метода векторного разрешения — графический метод (метод параллелограмма) и тригонометрический метод.

        Дополнительная практика Используйте компоненты виджета Vector ниже, чтобы разложить вектор на его компоненты. Просто введите величину и направление вектора.Затем нажмите кнопку Отправить , чтобы просмотреть горизонтальные и вертикальные компоненты. Используйте виджет как инструмент практики.

        Величина и направление вектора

        Онлайн-калькулятор для вычисления величины и направления вектора из его составляющих.

        Пусть v — вектор, заданный в компонентной форме формулой
        v = 1 , v 2 >
        Величина || v || вектора v задается формулой
        || v || = √ (v 1 2 + v 2 2 )
        , а направление вектора v — это угол θ в стандартном положении, такой что
        tan (θ) = v 2 / v 1 , так что 0 ≤ θ <2π.

        Использование калькулятора для расчета величины и направления

        1 — Введите компоненты v 1 и v 2 вектора v как действительные числа и нажмите «Рассчитать величину и направление». Выходы — величина || v || и направление θ в градусах вектора v.

        Используйте калькулятор величины и направления, чтобы ответить на вопросы

        1. Используйте калькулятор, чтобы найти направление векторов u = <- 2, 3> и v = <- 4, 6>.Почему они равны?
        2. Найдите направление векторов u = <2, 5> и v = <- 2, - 5>. Почему разница между двумя направлениями равна 180?
        3. Используйте калькулятор, чтобы найти направление векторов u = <2, 1> и v = <1, 2>. Почему сумма двух направлений равна 90? Найдите другие пары векторов, направления которых в сумме дают 90

        Дополнительные ссылки и ссылки

        Определение величины и направления векторов
        Векторные калькуляторы.
        Сложение векторов и скалярное умножение.

        Математические упражнения и математические задачи: векторы

        Найдите длину отрезка и координаты его средней точки. Нарисуйте отрезок в системе координат:

        Найдите координаты вектора, заданного двумя точками, и вычислите размер вектора.Нарисуйте вектор в системе координат:

        Найдите координаты векторов — v , 2 v , –2,5 v и нарисуйте векторы в системе координат, если:

        Выяснить, являются ли данные векторы линейно зависимыми (коллинеарными). Если да, найдите коэффициент коллинеарности k :

        Рассмотрим три вектора u = (–1; 5), v = (2.7; 3.8), w = (4,2; –6). Найдите координаты векторов:

        Найдите скалярное произведение векторов:

        Найдите размер угла между векторами:

        Узнать, перпендикулярны ли заданные векторы друг другу:

        Рассмотрим два вектора u = (3; –2), v = (–1; b ).Определите параметр b так, чтобы:

        Найдите вектор u , перпендикулярный вектору v = (3; 4) и размер которого равен 15.

        Докажите, что треугольник ABC , A [16; 1; –2], B [–9; 1; –2], C [0; 1; 10] прямоугольный. Найдите его периметр, площадь и размер внутренних углов.

        Рассмотрим три точки: A [0; 1; 2], B [1; 2; 0], C [2; 0; 1].


        a) Докажите, что точки A , B , C образуют треугольник.
        б) Найдите величину внутреннего угла α.
        c) Найдите длину медианы стороны a и координаты центра тяжести T .
        г) Найдите периметр треугольника ABC .
        e) Найдите площадь треугольника ABC .

    Добавить комментарий

    Ваш адрес email не будет опубликован. Обязательные поля помечены *